Aquifer Internal Medicine End of Case Questions

¡Supera tus tareas y exámenes ahora con Quizwiz!

A 72-year-old male presents with one hour of chest pain. The pain is vague, difficult to localize, gives a "squeezing" sensation, and is associated with dyspnea and nausea. On examination, his pulse is 110 beats/minute and regular, respiratory rate is 22 breaths/minute, blood pressure is 160/95 mmHg, and oxygen saturation is 92% on ambient air. He appears to be in moderate distress from pain. The rest of the exam is normal. What is the most likely diagnosis? A. Acute coronary syndrome B. Aortic dissection C. Pericarditis D. Pneumothorax E. Pulmonary embolism

The answer is A. From IM 01.

A 58-year-old female presents to the clinic with concern for chest pain over the past three months. She describes the pain as sharp and stabbing, in the mid-sternal region, lasting for one to two minutes, occurring a few times a day. The pain can come on at rest or with exertion and resolves on its own. It has not become worse since it began. There is no associated diaphoresis, shortness of breath, nausea, jaw pain, or pain with movement, eating, or laying supine. She has a 10-year history of obesity and hypertension for which she takes chlorthalidone and lisinopril. She was recently diagnosed with diabetes that has been controlled by diet. Physical examination shows her pulse is 86 beats/minute, respiration rate is 16 breaths/minute, and blood pressure is 135/85 mmHg. Her lungs are clear, heart sounds are normal, and there is no chest wall tenderness to palpation or abdominal tenderness. There is no peripheral edema. How would you best characterize her chest pain? A. Atypical angina B. Gastroesophageal reflux C. Musculoskeletal D. Stable angina E. Unstable angina

The answer is A. From IM 02.

A 66-year-old male presents to his primary care provider with concern for intermittent chest pain. He describes several episodes a week of pressure under his sternum that comes on with exertion, lasts for about 5 to 10 minutes and is improved with rest, over the last two weeks. He notes some shortness of breath with the pain, but denies any associated diaphoresis, nausea, or jaw pain. He has a history of hypertension and hyperlipidemia treated with lisinopril, simvastatin, and aspirin. He exercises at a gym for 30 minutes, two to three times per week. Physical examination shows his pulse is 78 beats/minute, respiratory rate is 16 breaths/minute, and blood pressure is 145/80 mmHg. His lungs are clear, heart sounds are normal, and there is no lower-extremity edema. His electrocardiogram (ECG) is normal. Which of the following is the most-appropriate next step in his workup? A. Cardiac catheterization B. Computed tomography (CT) scan C. Echocardiogram D. Pharmacologic stress test with imaging E. Treadmill stress test without imaging

The answer is A. From IM 02. The patient should proceed directly for a cardiac catheterization. His symptoms meet the criteria for angina (substernal chest discomfort with a characteristic duration and features, exertional in nature and relieved with rest or nitroglycerin) and since it is relatively new and started within the month, it should be considered unstable angina. In addition, he has considerable risk factors, thus his pretest probability is high for cardiac disease and cardiac catheterization is recommended.

A 68-year-old male with hypertension and non-ischemic cardiomyopathy is admitted to the hospital with an acute exacerbation of his congestive heart failure (CHF). An echocardiogram (ECG) one month ago showed left-ventricular hypertrophy and a left-ventricular ejection fracture (LVEF) of 60%. He takes ramipril and chlorthalidone at home, and these are continued. His pulse is 85 beats/minutes, blood pressure is 134/88 mmHg, and oxygen saturation is 95% on two liters of oxygen by nasal cannula. His exam is notable for jugular venous pulse and pressure (JVP) of 12, bilateral leg edema, and crackles at his lung bases. What will be the next step in management of this man? A. Add furosemide intravenously B. Add metoprolol orally C. Add valsartan D. Increase dose of chlorthalidone E. Increase dose of ramipril

The answer is A. From IM 04.

Which of the following physical examination findings is most consistent with right-sided heart failure? A. Abdominojugular reflux B. Bibasilar crackles C. Pitting pedal edema D. Sustained apical impulse E. Systolic murmur at right upper sternal border

The answer is A. From IM 04.

A 23-year-old female with diabetes mellitus type 1, hypertension, and hypothyroidism is admitted to the intensive care unit (ICU) with lethargy, nausea, and diarrhea. She is somnolent on admission, however, her boyfriend provides some history. She has not taken her insulin for the past three days due to nausea and vomiting, she has had a poor appetite and diarrhea. She is difficult to arouse. On exam, she is afebrile. She has no other neurologic deficits. Her admission labs are shown: Sodium: 123 mEq/ Potassium: 4.8 mEq/L Chloride: 91 mEq/L Bicarbonate (HCO3): 10 mEq/L Blood urea nitrogen (BUN): 48 mg/dL Creatinine (Cr): 1.5 mg/dL Serum glucose: 540 mg/dL White blood cell (WBC): 8 cells x 103/μL Serum osmolality: 285 mosm/kg Urinalysis is pending. What is the most likely cause of her clinical presentation? A. Diabetic ketoacidosis B. Diarrhea C. Pseudohyponatremia D. Sepsis with lactic acidosis E. Surreptitious consumption of isopropanolol

The answer is A. From IM 07.

A 62-year-old male presents to his physician for a follow-up of his type 2 diabetes mellitus. He is asymptomatic and is currently taking metformin and aspirin. His pulse is 76 beats/minute, blood pressure is 142/92 mmHg, and body mass index (BMI) is 29 kg/m2. The remainder of his physical examination is within normal limits. Laboratory results are shown as: Glucose 132 mg/dL Urinalysis: Negative leukocyte esterase, negative nitrite, negative bili, negative blood, trace protein, Microalbumin 275 mcg/g. Initiation of which a drug from which of the following classes would be most appropriate for this patient? A. Angiotensin converting enzyme inhibitor B. Beta-blocker C. Calcium channel blocker D. Thiazide diuretic E. Vasodilator

The answer is A. From IM 08.

A 36-year-old female is receiving a blood transfusion after having an abdominal hysterectomy. She has never had a blood transfusion previously. Both she and donor blood are type A+. As the first of two ordered units of packed red blood cells (PRBCs) is being infused, she develops fever and rigors. On exam, temperature is 37.9C (100.2F), pulse is 95 beats/minute, blood pressure is 122/68 mmHg, and oxygen saturation is 97% on room air. She appears diaphoretic. Lungs are clear with non-labored breathing. Abdominal exam is benign. There is yellow urine in her Foley catheter bag. Laboratory testing reveals hemoglobin of 6.5 g/dL, up from 6.0 g/dL postoperatively. Repeat blood type and crossmatch shows no incompatibility. What is the next best step in management for this patient? A. Use leukoreduced blood products for future transfusions B. Administer empiric broad-spectrum antibiotics C. Administer intramuscular (IM) epinephrine D. Give irradiated blood products for the next transfusion E. Give washed red blood cells for the next transfusion

The answer is A. From IM 10.

A 68-year-old male with a history of peptic ulcer disease, diabetes mellitus, and hypertension is admitted to the hospital with a 24-hour history of abdominal pain and three episodes of loose stool. Initial vital signs reveal temperature is 38.9 C (102.02 F), pulse is 102 beats/minute, respiratory rate is 16 breaths/minute, blood pressure is 132/78 mmHg, and oxygen saturation is 98% on room air. White blood count (WBC) is 13,100 cells/μl. He has tenderness to palpation of the left lower quadrant of his abdomen, with guarding but no rigidity. Renal function is normal. Computerized tomography (CT) scan of the abdomen and pelvis shows sigmoid diverticulitis without any sign of abscess or perforation. In addition to ordering intravenous (IV) antibiotics, IV fluid, and pain medication, what is the next best step in management? A. Blood cultures B. Colorectal surgery consult C. High-fiber diet D. Stool culture E. Stool test for occult blood

The answer is A. From IM 12.

A 51-year-old healthy male comes to the clinic to review the findings from his recent screening colonoscopy. He had multiple diverticula in the sigmoid colon and internal hemorrhoids, but no colonic polyps. Family history is negative for colon cancer. Physical examination is normal. Which one of the following recommendations for future colorectal cancer screening would be most appropriate for this patient? A. Colonoscopy in 10 years for secondary prevention B. Fecal immunochemical testing in five years for secondary prevention C. Flexible sigmoidoscopy in five years for tertiary prevention D. Yearly fecal occult blood for tertiary prevention E. Yearly flexible sigmoidoscopy for primary prevention

The answer is A. From IM 13.

A 52-year-old female presents for a follow-up visit for hypertension. Her aunt recently died from breast cancer, her mother is alive and without a history of cancer, and her father died 10 years ago from prostate cancer. The patient does not smoke cigarettes. She exercises regularly. Vital signs are pulse is 76 beats/minute, blood pressure is 138/72 mmHg, and body mass index (BMI) is 24.6 kg/m2. Her examination shows no abnormalities. Which U.S. Preventive Services Task Force (USPSTF) measures would you recommend to her to decrease her risk of breast cancer death? A. Biennial screening mammography B. Breast magnetic resonance (MR) now, then yearly screening mammography C. Clinical breast examination every six months by a clinician D. Monthly breast self-examination E. Yearly screening mammography

The answer is A. From IM 13.

A 61-year-old male with a history of chronic obstructive pulmonary disease (COPD) is seen for a follow-up visit. He has smoked one pack per day for 45 years but recently cut back to half-a-pack per day after a recent upper respiratory infection. On examination, vital signs are within normal ranges. He has mild diffuse wheezing on lung exam, and the remainder of his exam is normal. What is the most appropriate next step in managing his nicotine dependence? A. Assess his readiness to quit smoking B. Prescribe a long-acting anticholinergic inhaler C. Prescribe nicotine replacement therapy D. Prescribe varenicline E. Refer for smoking cessation counseling

The answer is A. From IM 15.

A 52-year-old female comes to the clinic to discuss weight loss. Her medical history is significant for obesity; her body mass index (BMI) is 41; hypertension; hyperlipidemia; and obstructive sleep apnea. She knows that losing weight will help her hypertension and hyperlipidemia, but she doesn't feel like these things bother her. Her only other concern is fatigue; she doesn't use her continuous positive airway pressure (CPAP) machine, because she doesn't like the mask. What additional information can you provide her to help motivate her weight loss? A. Her obstructive sleep apnea may improve with weight loss. B. Her risk of cardiovascular disease is similar to that of a female with a normal BMI. C. Obesity is mainly a cosmetic issue. D. Surgery should be considered before diet and exercise.

The answer is A. From IM 16.

A 67-year-old male comes to the clinic for a health maintenance visit. His past medical history is significant for chronic allergic rhinitis, severe chronic obstructive pulmonary disease (COPD), osteoporosis, psoriasis, atrial fibrillation, and benign prostatic hypertrophy. Vital signs show his temperature is 36.8 C (98.2 F), pulse is 76 beats/minute, respiratory rate is 12 breaths/minute, and blood pressure is 118/70 mmHg. His weight is 129.2 kg (285 lbs) and his body mass index (BMI) is 41. Which of his co-morbidities is most likely to be associated with his BMI? A. Atrial fibrillation B. Benign prostatic hypertrophy C. Chronic allergic rhinitis D. Osteoporosis E. Psoriasis

The answer is A. From IM 16.

You are asked to evaluate a 75-year-old female with a history of coronary artery disease and hypertension who presents with forgetfulness and difficulty caring for herself. She is accompanied by her son who has noticed a progressive functional decline in his mother over at least the past year. She has been increasingly forgetful, including several incidents wherein she would leave water for tea boiling for hours. In the past two months, he has taken over the management of her bills, as she had missed several payments. She is able to ambulate without assistance. Two days ago, her neighbors found her wandering around the neighborhood in the middle of the night. When they tried to redirect her, she became agitated, yelling that strangers were trying to rob her. She calmed down when her son arrived to the scene. She states that she does not feel depressed. Her vital signs are normal. Her physical exam was unremarkable and a detailed neurological exam demonstrates no significant abnormal findings except for a Montreal Cognitive Assessment (MoCA) score of 10. She completes a "Get Up and Go" test in 15 seconds, rising from the chair and ambulating without difficulty. A comprehensive metabolic profile, complete blood count, thyroid stimulating hormone, and vitamin B12 level are normal. A non-contrast computed tomography (CT) of her head shows increased sulci. No other structural abnormalities are seen. What is the most-likely diagnosis? A. Alzheimer's dementia B. Frontotemporal dementia (FTD) C. Lewy body dementia D. Pseudodementia secondary to depression E. Vascular dementia

The answer is A. From IM 18.

A 22-year-old male comes to your office with fatigue and shortness of breath on exertion. On exam, you notice conjunctival pallor. You obtain a hemogram, which shows the following: Hemoglobin: 10 g/dL Hematocrit: 29% Mean corpuscular volume (MCV): 74 μm3 Red blood cell distribution width (RDW): 14% What is the most likely diagnosis? A. Beta thalassemia B. Hemolytic anemia C. Hypothyroidism D. Iron deficiency anemia E. Sideroblastic anemia

The answer is A. From IM 19.

A 53-year-old female comes to the clinic to follow up on blood work obtained last week. Due to signs and symptoms of anemia, a complete blood count was ordered. Results are shown.White blood count (WBC): 7.4 cells x 103/μL Hemoglobin: 9 g/dL Hematocrit: 27% Mean corpuscular volume (MCV): 75 μm3 Platelets: 338,000 /mm3. Which test would be most helpful to diagnose iron deficiency in this patient? A. Ferritin B. Reticulocyte count C. Serum iron D. Total iron binding capacity E. Transferrin saturation

The answer is A. From IM 19.

A 56-year-old female is being evaluated in the hospital for anemia. She was admitted five days ago with a myocardial infarction. She underwent stent placement and was doing well until last night when she developed lower abdominal cramping and started passing blood clots in her stools. Her medical history is significant for type II diabetes mellitus, hypertension, hypothyroidism, and rheumatoid arthritis. On physical examination, she appears to be in mild distress. Her pulse is 100 beats/minute and blood pressure is 110/54 mmHg. Abdominal exam reveals tenderness in bilateral lower quadrants with no guarding or rigidity. The remainder of her exam is normal. A complete blood count (CBC) obtained today is shown. White blood count (WBC): 5 cells x103/μL Hemoglobin (Hgb): 8 g/dLHematocrit (Hct): 24% Mean corpuscular volume (MCV): 84 μm3 Red blood cell distribution width (RDW): 12 Platelets: 280,000 /mm3A complete blood count obtained at the time of hospital admission is shown. WBC:11 cells 103/μL Hgb:13 g/dL Hct: 39% MCV: 86 μm3 RDW: 11 Platelets:150,000 /mm3 What is the most likely cause of this patient's anemia? A. Acute blood loss B. Anemia of chronic disease C. Hemolysis D. Hypothyroidism E. Iron deficiency

The answer is A. From IM 19.

A female with human immunodeficiency virus (HIV) and a CD4 count of 24 cells/mm3 presents with fever and diarrhea for two months. She states no blood or mucus in her stool. She has not had any problems with diarrhea in the past. She reports no change in her diet. She does not take any medications. Which of the following would be the most likely diagnosis in this patient? A. Cytomegalovirus (CMV)-associated colitis B. E. coli O157:H7 infection C. Gluten intolerance D. Lymphocytic colitis E. Ulcerative colitis

The answer is A. From IM 20.

A 36-year-old female without any past medical history presents to the emergency department with severe dehydration. Which of the following urinalysis results would be most likely found in this patient? A. Decreased urine sodium (Na), decreased fractional excretion of sodium (FENa), increased urine osmolality B. Decreased urine sodium (Na), increased FENa, decreased urine osmolality C. Increased urine Na, decreased FENa, increased urine osmolality D. Increased urine Na, increased FENa, decreased urine osmolality

The answer is A. From IM 21.

A 63-year-old male with no past medical history is hospitalized after a fall and inability to care for himself at home. On his fourth day in the hospital, he is transferred to the intensive care unit (ICU) for sepsis due to a urinary tract infection (UTI). Which of the following would be the most-appropriate empiric antibiotic treatment for this patient? A. Ampicillin and gentamicin B. Azithromycin and trimethoprim-sulfamethoxazole C. Ceftriaxone and cefipime D. Ciprofloxacin and nitrofurantoin E. Vancomycin and metronidazole

The answer is A. From IM 21.

A 58-year-old female with chronic kidney disease stage III, secondary to type II diabetes mellitus, presents to the clinic to establish care. Her blood pressure is 154/86 mmHg. Her body mass index (BMI) is 30 kg/m2. Her hemoglobin A1C is 9.8 mg/dL, and urine protein/creatinine ratio is 1.5 mg/dL. She takes long-acting insulin. In addition to tighter glucose management, which of the following would be the best treatment for diabetic nephropathy? A. Lisinopril B. Pentoxifylline C. Protein restriction D. Simvastatin E. Spironolactone

The answer is A. From IM 23.

A 75-year-old male comes to the clinic for a review of the workup of his chronic kidney disease (CKD). His past medical history is significant for hypertension, diet-controlled diabetes mellitus type II, coronary artery disease, and benign prostatic hyperplasia (BPH). His blood pressure and diabetes have been well-controlled by antihypertensive medications and following a diabetic diet. Urinalysis and urine microscopy are normal. His creatinine is 1.5 mg/dL with a GFR of 55mL/min. Renal ultrasound shows normal-sized kidneys with good corticomedullary differentiation, mild hydronephrosis, and normal blood flow per dopplers. What is the most likely cause of his chronic kidney disease? A. Benign prostatic hyperplasia B. Diabetes mellitus C. Glomerulonephritis D. Hypertension E. Renal artery stenosis

The answer is A. From IM 23.

A 61-year-old male recently diagnosed with small cell lung carcinoma presents with confusion. Head imaging is negative for brain metastases, but his serum sodium is 120 mEq/L. You know that syndrome of inappropriate antidiuretic hormone (SIADH) secretion is a common paraneoplastic syndrome associated with small cell lung carcinoma. If SIADH is causing his hyponatremia, what results would you expect on his clinical and laboratory evaluations? A. Euvolemia/low serum osmolarity/high urine osmolarity B. Hypervolemia/high serum osmolarity/low urine osmolarity C. Hypervolemia/low serum osmolarity/high urine osmolarity D. Hypovolemia/high serum osmolarity/low urine osmolarity E. Hypovolemia/low serum osmolarity/high urine osmolarity

The answer is A. From IM 25.

A 23-year-old male is brought to the hospital on a hot summer evening with an altered mental status. He has no chronic medical conditions and takes no prescribed medications. He is known to drink alcohol regularly and use street drugs. He reportedly attended a party hours before being found confused and combative in his apartment, which is without air conditioning. Laboratory examination reveals a serum sodium of 160 and a creatinine of 2.1 mg/dL. What is the most likely cause of his hypernatremia? A. Alcohol intoxication with vomiting B. Diuresis from excessive beer intake C. Excess salty food intake D. Heroin overdose E. MDMA ("ecstasy") intoxication

The answer is A. From IM 26.

A 58-year-old male experiencing homelessness presents to the emergency department with a decreased level of consciousness. Urine microscopy is performed, and numerous calcium oxalate crystals are present. What is the most likely explanation for the calcium oxalate crystals seen in his urine? A. Ethylene glycol ingestion B. Excessive ethanol intake C. Kidney stones D. Methanol poisoning E. Tricyclic antidepressant overdose

The answer is A. From IM 26.

While on call in the hospital, you are called to the bedside of a female who was admitted four hours ago with severe pneumonia. Her breathing has become more shallow, she is increasingly lethargic and her oxygen saturation has dropped to 85% despite high-flow oxygen. You request an arterial blood gas (ABG) to help determine your next steps. Which of the following ABG results is most consistent with this patient's clinical presentation? A. pH: 7.15p CO2: 70 mmHg pO2: 60 mmHg HCO3: 28 mEq/L B. pH: 7.30p CO2: 30 mmHg pO2: 90 mmHg HCO3: 12 mEq/L C. pH: 7.37p CO2: 40 mmHg pO2: 90 mmHg HCO3: 22 mEq/L D. pH: 7.50p CO2: 60 mmHg pO2: 90 mmHg HCO3: 44 mEq/L E. pH: 7.65p CO2: 20 mmHg pO2: 70 mmHg HCO3: 16 mEq/L

The answer is A. From IM 28.

You are seeing a female in your clinic with unilateral leg swelling and pain as her chief concern. On exam, she has right-lower leg tenderness, edema, erythema, and a positive Homan's sign (calf pain with passive dorsiflexion of the ankle). Of her signs and symptoms, which, if any, is the most sensitive and specific for deep-vein thrombosis (DVT)? A. None B. Pitting lower-extremity edema C. Positive Homan's sign D. Tenderness on physical exam E. Unilateral leg pain

The answer is A. From IM 30.

A 20-year old male presents to the internal medicine clinic with concerns of right knee pain and an inability to bear weight. He states that he was practicing speed drills (accelerating, decelerating, and changing directions), when he heard a "pop" in his right knee followed by immediate right knee instability and swelling. What is the MOST likely diagnosis? A. Anterior cruciate ligament (ACL) tear B. Lateral collateral ligament (LCL) tear C. Medial collateral ligament (MCL) tear D. Patellar subluxation E. Posterior cruciate ligament (PCL) tear

The answer is A. From IM 31.

You are seeing a 45-year-old female in the primary care clinic with concerns of joint pain. She reports a six-month history of symmetrical pain and swelling of her metacarpophalangeal (MCP) and proximal interphalangeal (PIP) joints of both hands as well as her wrists, which are more painful and stiff in the morning. She also endorses difficulty holding her coffee mug due to weakness and generalized fatigue. On exam, you notice tenderness and edema of the wrists, MCPs and PIPs bilaterally. There are no oral lesions, she has no skin changes, no muscle tenderness, and no other weakness appreciated. Her exam is otherwise normal. Which of the following would be the most-appropriate diagnostic laboratory test to order? A. Anti-citrullinated peptide antibodies (anti-CCP) B. Anti-double stranded DNA (anti-dsDNA) C. Anti-neutrophil cytoplasmic antibodies (ANCA) D. Antinuclear antibody (ANA) E. Erythrocyte sedimentation rate (ESR)

The answer is A. From IM 32.

In the emergency department, you are evaluating a 64 year old previously healthy male presenting with fever, altered mental status and shortness of breath. At admission, his temperature is 103 degrees Fahrenheit, heart rate is 128 beats per minute, respiratory rate is 27 breaths per minute, oxygen saturation 94% on 15L non-rebreather. On exam, he is hypotensive and appears acutely ill, with somnolence and respiratory distress with diffuse crackles. While your attending prepares to intubate the patient, bloodwork is collected. Labs show the following: Na 134 mEq/L Cl 100 mmol/L HCO3 14 mmol/L Lactic acid 4.9 mmol/L ABG: pH 7.33 / PaO2 61 mm Hg / PaCO2 31 mm Hg How do you interpret his acid/base status? A. Anion gap metabolic acidosis B. Anion gap metabolic alkalosis C. Anion gap respiratory alkalosis D. Non-anion gap metabolic acidosis E. Non-anion gap metabolic alkalosis

The answer is A. From IM 33.

You are working at a skilled nursing facility, rounding on your patients. The nurse tells you that Mrs. Viera, an 83 year-old resident with a history of heart failure, has been having vomiting, diarrhea, and fever since the previous afternoon. The nurse also reports a decrease in urine output since early that morning. On your assessment, you notice that Mrs. Viera is slightly more confused than baseline, with dry mucous membranes. On chart review, you see that her baseline creatinine is 1.6 mg/dL, and that her medication list includes furosemide. You order stat labs which come back with the following results: Serum sodium: 134 mEq/L Serum blood urea nitrogen (BUN): 17 mg/dL Serum creatinine: 2.1 mg/dL Urinary sodium: 200 mEq/L Urinary blood urea nitrogen (BUN): 385 mg/dL Urinary creatinine: 220 mg/dL How does the fractional excretion of sodium (FENa) help you identify the etiology of the patient's acute kidney injury (AKI)? A. FENa cannot be reliably interpreted in this patient B. FENa is <1%, indicating a pre-renal etiology C. FENa is <1%, indicating likely acute tubular necrosis (ATN) D. FENa is >1%, indicating a pre-renal etiology E. FENa is >1%, indicating likely acute tubular necrosis (ATN)

The answer is A. From IM 33.

A 48-year-old male with no significant past medical history presents to your office in the summer with eight days of fever, sore throat, rash, and myalgias. His nine-year-old daughter was treated for strep throat recently. He is a non-smoker, drinks four to five alcoholic drinks on the weekends, and does not use intravenous drug use (IVDU). He is sexually active with women. Exam results show temperature is 39.4 C (103 F), pulse is 116 beats/minute, blood pressure is 120/72 mmHg. He is ill-appearing, and your exam confirms swollen and erythematous pharynx, palpable and tender cervical and axillary lymph nodes, a diffuse maculopapular rash, and a palpable spleen. Examination of his genitalia is normal. Which of the following is the next best step in evaluating and managing this patient? A. Obtain a human immunodeficiency virus (HIV) viral load B. Obtain HIV antibody testing C. Perform a rapid strep test (RST) D. Perform darkfield microscopy on the rash E. Recommend hydration and rest

The answer is A. From IM 35.

Mr. York is a 44-year-old male presenting for evaluation of an eyelid lesion. He noticed the lesion about one year ago. There is no associated itching, discharge, or other bothersome symptoms. Which of the following is the next best step in the management of the eyelid lesion? A. Low potency topical corticosteroid B. Measurement of serum cholesterol levels C. Measurement of serum uric acid levels D. No further management E. Skin biopsy

The answer is B. From IM !6.

A 42-year-old female presents to the emergency department with concern for mild chest pain lasting three to four minutes with vigorous exercise, three times over the past week. She has no past medical history and is not taking any medications or supplements. She has no family history of cardiac or pulmonary disorders. She follows a vegetarian diet, exercises regularly, and is training for a half-marathon. Physical examination shows her pulse is 66 beats/minute, respiration rate is 16 breaths/minute, and blood pressure is 110/70 mmHg. Her lungs are clear, heart sounds are normal, and there is no lower-extremity edema. Which of the following is the most likely laboratory study in the acute setting to assist with the diagnosis? A. Brain natriuretic peptide (BNP) B. Complete blood count (CBC) C. Hemoglobin A1c D. Low-density lipoprotein (LDL) E. Triglycerides

The answer is B. From IM 02. Although cardiac causes of chest pain should be considered, it is important to consider other sources of pain. Anemia may cause chest pain by decreasing oxygen carrying capacity.

Mary is an 18-year-old female with a past medical history significant for type 1 diabetes mellitus who was admitted to the intensive care unit (ICU) yesterday with nausea, vomiting, and a decreased level of consciousness. Her labs were consistent with diabetic ketoacidosis, and she was started on an insulin drip and intravenous fluids. Today, she is more alert and has no further nausea or vomiting. Her anion gap has closed. Prior to stopping the insulin drip, which type of insulin is most likely to prevent a recurrence of ketoacidosis? A. Intermediate-acting insulin B. Long-acting insulin C. Premixed insulin (combination of short- and long-acting insulin) D. Rapid-acting insulin E. Short-acting insulin

The answer is B. From IM 07.

A 55-year-old male with hypertension and type II diabetes mellitus presents to the office for a routine follow-up. He has a history of bladder cancer that was treated by resection and chemotherapy five years ago. He is currently asymptomatic. He takes lisinopril, aspirin, and metformin. His pulse is 76 beats/minute, blood pressure is 128/78 mmHg, and body mass index (BMI) is 29 kg/m2. The rest of his physical exam is unremarkable. Laboratory studies reveal: Glucose 142 mg/dL Urinalysis: Negative leukocyte esterase, negative nitrite, negative bili, negative blood, trace protein Microalbumin 275 mcg/g Lipid Profile: Total cholesterol - 295, HDL - 35, LDL - 220, Triglycerides - 200 HgbA1c 7.2% Which of the following is the most appropriate medication to initiate in this patient? A. Amlodipine B. Atorvastatin C. Losartan D. Niacin E. Pioglitazone

The answer is B. From IM 08.

A 24-year-old male comes to the clinic to establish care as a new patient. His medical history is significant for intermittent asthma. Social history reveals that he is a graduate student. He says that he drank alcohol heavily when he was in an undergraduate fraternity two years ago, but that he has cut back significantly on his alcohol use and no longer drinks every day. He does have intense cravings for alcohol at times and has missed a few classes because he was hungover from a binge and unable to get to class. He received a driving-under-the-influence (DUI) citation two months ago, which he admits was a "really stupid mistake." On physical exam, he is well-groomed and pleasant. His vital signs reveal pulse is 80 beats/minute, respiration rate is 12 breaths/minute, blood pressure is 138/82 mmHg. His physical examination is normal. What is the best description of the alcohol consumption in this patient? A. Alcohol abuse B. Alcohol use disorder C. Occasional alcohol use D. Previous alcohol dependence E. Risky alcohol behavior

The answer is B. From IM 09.

A 50-year-old male with history of alcohol use disorder, type 2 diabetes mellitus, and cirrhosis presents to the emergency department with hematochezia and lightheadedness. He reports a large amount of painless bright-red rectal bleeding for the past one hour. He has not experienced this before. He denies vomiting. His home medications include metformin and propranolol. He drinks half a pint of vodka daily. On physical exam, his temperature is 37.3C (99.2F), pulse is 120 beats/minute, and blood pressure is 82/60 mmHg. His abdomen is soft and non-tender. He is jaundiced and has distended veins on his anterior abdominal wall. Laboratory testing reveals a hemoglobin of 8.8 mg/dL and platelet count of 69,000/µL. Two large-bore peripheral intravenous (IV) medications are placed, a fluid bolus is started, and a blood type and crossmatch is ordered. What diagnosis is the most likely cause of this patient's hematochezia? A. Alcoholic gastritis B. Esophageal varices C. Internal hemorrhoids D. Mallory-Weiss tear E. Meckel diverticulum

The answer is B. From IM 10.

Six weeks ago, a 67-year-old female presented to her primary care physician with dyspepsia and epigastric pain. She did not drink alcohol and took no medications. Testing revealed a positive stool-occult blood test and mild microcytic anemia. She underwent upper and lower endoscopy and was found to have a duodenal ulcer. Biopsy was positive for Helicobacter pylori (H. pylori), and she completed the prescribed course of proton-pump inhibitor and antibiotics. Now she is returning to the clinic and reports that her symptoms have not resolved. What is the best next step in diagnosis and management for this patient? A. Perform computed tomography (CT) of the abdomen B. Perform H. pylori stool antigen test C. Repeat upper endoscopy with biopsy D. Resume proton-pump inhibitor indefinitely E. Test serum gastrin level

The answer is B. From IM 10.

Two victims of a multiple-vehicle car accident are being treated in the emergency department trauma bay. Both have severe injuries resulting in blood loss, and four units of packed red blood cells (PRBCs) are ordered from the blood bank for each patient. The stickers to label each patient's blood samples were mixed up, so these two patients each mistakenly receive the blood intended for the other. The patient with type O+ blood ends up receiving a transfusion of B+ blood, and you are concerned that she will develop an acute hemolytic transfusion reaction. What findings would be most consistent with this type of transfusion reaction? A. Fever and nausea within hours of the transfusion, with normal urine output B. Fever, dyspnea, hypotension, and dark urine within minutes of starting the transfusion C. Fever, spherocytosis, and hyperbilirubinemia 2 to 10 days after the transfusion D. Hypotension and angioedema within minutes of starting transfusion E. Pulmonary edema within hours of transfusion

The answer is B. From IM 10.

A 39-year-old male with no significant past medical history presents to the hospital with lower abdominal pain for the past two days. His initial vitals are notable where his temperature is 38.5 C (101.3 F), pulse is 112 beats/minute, and blood pressure is 103/68 mmHg. He reports that his last bowel movement was two days ago, and he recalls passing flatus earlier today. Your physical exam reveals hypoactive bowel sounds, and a diffusely tender abdomen, most tender in the right-lower quadrant. His abdominal muscles are tense, and do not relax with distraction techniques. A computed tomography (CT) scan of the abdomen and pelvis confirms your suspected diagnosis. You order broad-spectrum intravenous (IV) antibiotics. What is the best next step in management? A. Call a gastrointestinal (GI) consult for colonoscopy B. Call a surgical consult C. Order a bowel regimen D. Place a nasogastric tube for decompression E. Start IV steroids

The answer is B. From IM 12.

A 67-year-old female with a history of diet-controlled gastroesophageal reflux disease presents to her general internist for her annual well visit. She has been doing well since her last visit one year ago. Her only medication is omeprazole as needed, which she takes rarely. Six months ago, she had an unremarkable mammogram and four years ago she had an unremarkable colonoscopy. She has a history of regular cervical cancer screenings without any abnormal results. She exercises regularly, does not drink, and has never smoked. She works as a part-time librarian. Her mother and father both lived into their 80s, and both died of cardiac disease. Physical exam, including vital signs, are normal. Which of the following screening modalities is most appropriate for this patient? A. Abdominal ultrasound B. Dual-energy x-ray absorptiometry (DEXA) scan C. Papanicolaou (pap) test D. Transvaginal ultrasound E. Upper endoscopy

The answer is B. From IM 14.

A 44-year-old male presents with a three-week history of nasal congestion, rhinorrhea, and watery eyes. He notes an occasional dry cough and mild sore throat as well. On physical examination, vital signs are within normal limits. Nasal exam reveals mild mucosal edema that is boggy and pale. Pharynx has a cobblestoning appearance. What is the next best step in management? A. Antibiotic therapy B. Antihistamine therapy C. Complete blood count D. Rapid Strep test E. Sinus computed tomography (CT)

The answer is B. From IM 15.

A 57-year-old male presents with a five-day history of productive cough with purulent sputum, fatigue, and fever. He has a 40-pack-per-year smoking history. On examination, he is moderately ill-appearing. Temperature is 38.6 C (101.4 F), pulse is 110 beats/minute, respiratory rate is 22 breaths/minute, blood pressure is 118/76 mmHg, and oxygen saturation is 92% on room air. Chest exam reveals tachycardia without murmurs and crackles in the left-lower lung field. What is the most appropriate next diagnostic step? A. Chest computed tomography (CT) B. Chest x-ray C. Complete blood count (CBC) with differential D. Influenza testing E. Pulmonary function testing

The answer is B. From IM 15.

Ms. H is a 68-year-old female with a medical history significant for obesity, type II diabetes, hypothyroidism, hypertension, and recently diagnosed hyperlipidemia. Her most-recent lipoprotein (LDL), three months ago, was 197 mg/dL. At that time, atorvastatin was initiated. Other medications include metformin, insulin glargine, amlodipine, hydrochlorthiazide, and levothyroxine. Which of the following may be contributing to her elevated LDL? A. Amlodipine B. Hydrochlorthiazide C. Insulin glargine D. Levothyroxine E. Metformin

The answer is B. From IM 16.

A 28-year-old fair-skinned male presents with a skin lesion on his left upper arm that he is concerned about. The lesion has been present for as long as he can remember, however, just recently it has been changing in size and color. The lesion does not itch or cause pain. On physical exam, you note an 8 mm round symmetrical patch with areas of black. The lesion is flat without crusting or ulceration. What is the next step in his management? A. Deep-shave biopsy B. Excisional biopsy C. Monitoring of the lesion with follow-up in six months D. Punch biopsy E. Superficial shave biopsy

The answer is B. From IM 17.

A 30-year-old fair-skinned female presents to the office for her first skin check. She has multiple lesions on her back; each of them is slightly different. Which lesion would be most concerning for melanoma? A. A brown lesion measuring 4 mm of uncertain duration B. A symmetric black lesion which has doubled in size since childhood C. A uniformly black lesion present since childhood D. An asymmetric black lesion present since childhood E. An asymmetrical brown pigmented lesion present since childhood

The answer is B. From IM 17.

A 78-year-old male is evaluated in a subacute rehabilitation center after sustaining a fall. The fall occurred last night as he was getting out of bed. He does not recall the circumstances of the fall. Medical history is significant for Parkinson disease for which he has taken carbidopa-levodopa for four years. He was hospitalized for community-acquired pneumonia two weeks ago and completed his course of levofloxacin four days ago. During his admission, diphenhydramine was started and he continues to take it at bedtime. Today, his temperature is 37.6C (99.7F), pulse is 85 beats/minute, and blood pressure is 145/80 mmHg. His physical exam reveals lungs clear to auscultation, regular heart rate, no appreciable heart murmur, and mild ecchymosis on his left hip. He scores 27/30 on a mini-mental state examination (MMSE). He is able to stand and ambulate 20 feet without the use of assistance devices. He is cooperative during the exam. Which of the following most likely triggered this fall? A. Arrhythmia due to levofloxacin B. Delirium due to diphenhydramine C. Dementia secondary to Parkinson disease D. Limited mobility due to Parkinson disease E. Sepsis due to healthcare-associated pneumonia

The answer is B. From IM 18.

A 26-year-old male with human immunodeficiency virus (HIV) presents to the clinic with one week of blurry vision and floaters in his right eye. He has not had any exposure to bacterial conjunctivitis at his work in an elementary school. He does not wear corrective lenses. He had a fever this morning to 38.2 C (100.8 F) with associated malaise and mild abdominal pain and one watery stool. He admits to nonadherence with his HIV medication regimen. A funduscopic exam was performed, which showed the following. What is the most-likely organism responsible for these findings? Funduscopic findings A. Chlamydia trachomatis B. Cytomegalovirus C. Herpes zoster D. Retinitis pigmentosa E. Toxoplasmosis

The answer is B. From IM 20.

A 52-year-old bisexual male presents with difficulty swallowing solid foods for two weeks. He has also experienced low-grade fevers, weight loss, and anorexia. He admits to not consistently using condoms. He reports no vomiting, melena, or hematochezia. He takes no medications and does not smoke or drink alcohol. On exam, his oropharynx is normal. He has mild epigastric tenderness and appears cachectic. Laboratory evaluation reveals the following: White blood cell count (WBC): 3.5K (4.5-11); Hemoglobin: 10.2 gm/dl (12-15 gm/dl); Platelet count: 136K (150-420); Human immunodeficiency virus (HIV): PositiveCD4+ T-cell count: 188 cells/ml. An esophagogastroduodenoscopy (EGD) is ordered. What is the most likely finding on the EGD? A. Barrett's esophagus B. Esophageal candidiasis C. Gastritis D. Non-bleeding duodenal ulcer E. Normal

The answer is B. From IM 20.

A 36-year-old female presents to the emergency department with a one-day history of nausea, dysuria, and right-sided flank pain. She was previously healthy. Temperature is 39.3C (102.7F), pulse is 125 beats/minute, respiratory rate is 20 breaths/minute, and blood pressure is 78/50 mmHg. She receives four liters of normal saline and is started on intravenous (IV) vasopressors and antibiotics. A repeat blood pressure is 88/52 mmHg. What type of shock does this patient most likely have? A. Cardiogenic B. Distributive C. Hypovolemic D. Multifactorial E. Obstructive

The answer is B. From IM 21.

A 49-year-old female presents to the emergency department with a one-day history of bright-red blood per rectum. She reports no fevers, abdominal pain, or lightheadedness. Her medical history is significant for alcohol abuse and cirrhosis. She takes no medications. Her pulse is 90 beats/minute and blood pressure is 108/64 mmHg. On physical exam, she has bilateral lower-extremity edema and ascites. After fluid resuscitation and laboratory studies, which of the following is the most appropriate diagnostic study? A. Capsule endoscopy B. Colonoscopy C. Esophagogastroduodenoscopy (EGD) D. Radionuclide imaging E. Upper gastrointestinal (UGI) with small bowel follow-through

The answer is B. From IM 21.

A 72-year-old male presents to the hospital with a two-day history of bright-red blood per rectum. He reports no weight loss, fevers, chills, nausea, vomiting, or diarrhea. He has a history of hypertension, diabetes, hyperlipidemia, and chronic constipation. Temperature is 37.1C (98.8F), pulse is 98 beats/minute, respiratory rate is 20 breaths/minute, and blood pressure is 132/94 mmHg. Abdominal examination reveals slight tenderness in the left-lower quadrant, without guarding or rebound. The remainder of the physical examination is normal. Which of the following is the most likely diagnosis for this patient? A. Colon cancer B. Diverticulosis C. Gastritis D. Inflammatory bowel disease E. Perforated colon

The answer is B. From IM 21.

A 45-year-old female with stage 3 chronic kidney disease, secondary to lupus nephritis, presents to the emergency department with confusion. Her husband reports that she has been complaining of nausea and itching for several weeks. Diphenhydramine has not alleviated the itching. Her physical exam is significant for lethargy, orientation to person only, excoriations on all of her extremities, a pericardial friction rub, asterixis, crackles at the lower lung fields, and pitting edema of the lower extremities to the knees. Her labs are significant for sodium 135mEq/L, potassium 5.5mEq/L, creatinine 5mg/dL [creatinine was 2.5mg/dL six months ago], HCO3 20 mEq/L, and an anion gap of 14. What is the pathophysiological mechanism of her acute state? A. Decreased cardiac output from non-ischemic cardiomyopathy B. Inability of the kidney to excrete organic waste products C. Inability of the liver to excrete nitrogenous waste products D. Ingestion of ethanol, leading to accumulation of ketones E. Poor oral intake, leading to accumulation of ketones

The answer is B. From IM 23.

A 32-year-old male with type 1 diabetes mellitus presents to the emergency department. He has had recent suicidal ideation and recent admissions for diabetic ketoacidosis. He has a suspected, but not proven, ethylene glycol overdose, as his friend found an empty bottle of antifreeze in his house. His arterial pH is 7.25, his serum bicarbonate level is 14 mEq/L, and his osmolar gap is 12 mOsm/kg H2O. The ethylene glycol level is pending. What should be the immediate next step? A. Consult nephrology to initiate hemodialysis B. Order fomepizole C. Send urine to evaluate for calcium oxalate crystals D. Start an intravenous bicarbonate continuous infusion E. Start an intravenous insulin drip

The answer is B. From IM 26.

A 58-year-old female with a history of hypertension and hyperlipidemia on chlorthalidone and simvastatin presents to her physician with concerns of fatigue and constipation. Physical exam is significant for dry mucous membranes, her pulse is 105 beats/minute, and there is hard stool in the rectal vault. Her blood pressure is normal. Labs are significant for: glucose 100 mg/dL, sodium 140mEq/L, potassium 4.0mEq/L, calcium 12.0 mg/dL, thyroid-stimulating hormone (TSH) 3.5 mU/L, and parathyroid hormone (PTH) 18 pg/mL. What is the most likely cause of this patient's hypercalcemia? A. Adrenal insufficiency B. Chlorthalidone C. Hyperparathyroidism D. Hyperthyroidism E. Metastasis to the bone

The answer is B. From IM 27.

A 65-year-old female presents to the emergency department with one week of progressive confusion, fatigue, and global weakness. Her history is significant for widely metastatic renal-cell carcinoma. She has not had a bowel movement in five days. Laboratory studies are shown. Sodium: 142 mEq/L Potassium: 4.2 mEq/L Chloride: 105 mEq/L Bicarbonate: 18 mEq/L Calcium: 15 mg/dL Albumin: 3.9 g/dL Goals of care are discussed with her wife, and she requests inpatient therapy. Along with saline resuscitation, which medication should be given to help lower her calcium levels? A. Acetazolamide B. Calcitonin C. Furosemide D. Hydrocholorthiazide E. Spironolactone

The answer is B. From IM 27.

A 56-year-old female was diagnosed with left-lower-lobe pneumonia. She was hospitalized and has completed four days of ceftriaxone and azithromycin. She has a history of congestive heart failure (CHF), but was euvolemic on admission. You are evaluating her on hospital day five. She continues to complain of cough and dyspnea. Vital signs show temperature is 38.0 C (100.4 F), pulse is 90 beats/minute, respiratory rate is 24 breaths/minute, blood pressure is 122/78 mmHg, and oxygen saturation is 92% on four liters/minute. Her exam is significant for faint crackles and decreased breath sounds in the lower-left-lung fields. Her jugular venous pressure (JVP) is approximately 7 cm, and there is a trace of pedal edema bilaterally. A chest radiograph shows a stable left-lower-lobe infiltrate with a new, moderate left-pleural effusion. Which of the following is the appropriate next step in management? A. Chest computed tomography (CT) B. Diagnostic thoracentesis C. Electrocardiogram D. Repeat blood and urine cultures E. Transthoracic echocardiogram

The answer is B. From IM 29.

A 65-year-old male presents to the emergency department with dyspnea, cough, and fever for one week. His past medical history is significant for hypertension. On exam, his temperature is 38.5 C (101.3 F), pulse is 100 beats/minute, respiratory rate is 24 breaths/minute, blood pressure is 140/80 mmHg, and oxygen saturation is 88% on room air, increasing to 92% on four liters of oxygen by nasal cannula. The patient appears uncomfortable and is using accessory muscles of respiration. Breath sounds are decreased on the left side with dullness to percussion on the left. Laboratory values include a white blood cell (WBC) count of 15 cells x 103/μL hematocrit (Hct) of 45%, platelet count of 150,000/µL, and an international normalized ratio (INR) of 1.1. Chest radiograph shows a large left-sided pleural effusion. He is given broad-spectrum antibiotics. Which of the following is the most appropriate next step in management? A. Computed tomography (CT) scan of the chest B. Diagnostic and therapeutic thoracentesis C. Echocardiogram D. Intubation and mechanical ventilation E. Placement of a chest tube

The answer is B. From IM 29.

A 45-year-old previously healthy female presents to the emergency department with right-lower extremity swelling for two days. She has no chest pain or shortness of breath. Her vital signs are normal. Her exam is notable for swelling of the right thigh and calf; the rest of her exam is normal. Duplex ultrasound of the right-lower extremity reveals a popliteal deep-vein thrombosis (DVT) with extension above the popliteal fossa. Upon further discussion with her, you learn that she is visiting from another state and plans to stay in this area for at least a week. She has health insurance through her employer. What is the most-appropriate anticoagulation in this case? A. Discharge from the emergency department (ED), initiate low-molecular-weight heparin (LMWH) and warfarin B. Discharge from the ED, initiate oral apixaban C. Discharge from the ED, initiate warfarin without LMWH D. Hospitalize overnight, initiate heparin infusion and warfarin E. Hospitalize overnight, initiate LMWH and warfarin

The answer is B. From IM 30.

You are seeing a female in the emergency department with dyspnea and left-leg swelling. She is 48 years old with a history of deep-vein thrombosis (DVT) and pulmonary embolism (PE) three years ago. At that time, she was anticoagulated on warfarin for six months, and then it was discontinued. She reports having no fever, chills, or productive cough. On exam, her pulse is 110 beats/minute, respiratory rate is 24 breaths/minute, and the other vital signs are normal. Heart and lung exams are normal aside from tachycardia and tachypnea. There is swelling and tenderness of the left calf. What is the next appropriate diagnostic test? A. Chest radiograph B. Computed tomography angiogram (CTA) of chest C. High sensitivity D-dimer D. Lower extremity duplex ultrasound E. Ventilation-perfusion (V/Q) scan

The answer is B. From IM 30.

A 32-year-old female presents to the office with fever, a petechial skin rash, and migrating joint pain for the past month. She states that her symptoms started a few days after having an unprotected sexual encounter while on vacation. Two weeks ago, she had pain and erythema in her right wrist, which spontaneously improved. She now has pain in her left knee with an associated effusion. You perform an arthrocentesis in the office, which shows the following: Straw-colored, cloudy fluid low viscosity 25,000 white blood count (WBC)/mm 80% polymorphonuclear cells (PMNs) Gram stain and crystal evaluation is pending. What is the most likely diagnosis? A. Acute rheumatic fever B. Gonococcal arthritis C. Osteoarthritis D. Reactive arthritis E. Staphylococcal arthritis

The answer is B. From IM 31.

A 72-year-old male with a history of a right knee replacement 14 days ago returns to the emergency department with pain at the site of the knee replacement, and a new effusion. An arthrocentesis performed at his bedside reveals bloody fluid with low viscosity. Lab analysis reveals 5,000 white blood count (WBC)/mm with 30% polymorphonuclear cells (PMNs) and 900,000 red blood count (RBC)/mm in the fluid. What is the most-likely diagnosis? A. Gout B. Hemarthrosis C. Osteoarthritis D. Rheumatoid arthritis E. Septic arthritis

The answer is B. From IM 31.

You are working in the emergency department, and your attending asks you to see a 65-year-old male presenting with abdominal pain. In gathering the history, the patient tells you that he has had two to three days of progressive generalized "achy" abdominal pain, along with decreased appetite and decreased urine output. On further clarification, he reports not urinating in the past two days. On exam, vital signs are normal. He is alert, oriented, and in no acute distress. His exam is benign with the exception of mild abdominal distention, a palpable bladder, and an enlarged prostate on digital rectal exam. Which of the following is the next best step? A. Computed tomography (CT) of his abdomen and pelvis B. Foley catheter placement C. Intravenous (IV) diuretics D. IV fluids E. Renal ultrasound

The answer is B. From IM 33.

A 30-year-old female presents with an acute exacerbation of chronic lower back pain that has been present for the past six years. She reports no recent fevers or weight changes. She is also treated for generalized anxiety disorder but has no other chronic conditions. She takes ibuprofen for her pain and escitalopram for anxiety. Temperature is 37 C (98.6 F), pulse is 78 beats/minute, respiratory rate is 14 breaths/minute, blood pressure is 126/78 mmHg. Physical examination reveals a limitation of flexion and extension of the lumbar spine. Straight-leg raise test is negative. Her motor strength and sensory exam in both low extremities are normal. Her Achilles and patellar reflexes are 2+ bilaterally. The remainder of her examination is normal. Into which of the following diagnostic categories is her back pain most likely to fall? A. Infectious etiology B. Inflammatory process C. Metastatic malignancy D. Neurodegenerative disorder E. Psychiatric condition

The answer is B. From IM 34.

xA 42-year-old male presents to the office with concerns of low back pain and lower extremity weakness. He has no significant medical history and his only medication is occasional ibuprofen for pain. He is very active and has recently joined a recreational golf league. This current pain has persisted since his last golf game. Physical exam reveals a fit male in no distress with vitals showing his pulse is 56 beats/minute, respiratory rate is 16 breaths/minute, and blood pressure is 110/75 mmHg. You are preparing to perform a neurologic examination to diagnose what is likely a radicular etiology of his back pain. Which combination of motor, sensory, and reflex neurologic deficits would be consistent with an L5-S1 radiculopathy? A. Big toe dorsiflexion difficulty, third metatarsal sensory deficits, and decreased medial hamstring change B. Foot eversion difficulty, lateral malleolus sensory deficits, and decreased Achilles reflex C. Foot eversion difficulty, medial malleolus sensory deficits, and decreased Achilles reflex D. Foot inversion difficulty, lateral malleolus sensory deficits, and decreased patellar reflex E. Foot inversion difficulty, medial malleolus sensory deficits, and decreased patellar reflex

The answer is B. From IM 34.

You are taking care of a 63-year-old female on the internal medicine floor who was recently diagnosed with cirrhosis. She was admitted with hepatic encephalopathy which has improved with lactulose. Today when you see her, she notes that she has been feeling more fatigued and weak with some light-headedness upon standing. On exam, she is afebrile. Her pulse is 122 beats/minutes (increased from 80s previously), and her blood pressure is 90/58 mmHg (near baseline since admission). She is breathing comfortably with a normal oxygen saturation. Her cardiac exam is normal with the exception of regular tachycardia. Her abdomen is moderately distended with a positive fluid wave, non-tender, and with normal bowel sounds. She has bilateral 2+ pitting edema to the mid-thigh. What is the most appropriate next step in diagnosis? A. Blood cultures B. Complete blood count (CBC) C. Esophagogastroduodenoscopy (EGD) D. Liver biopsy E. Paracentesis

The answer is B. From IM 36.

A 68-year-old male presents with concern for substernal chest pressure with exertion lasting five minutes and alleviated with rest. He has a remote history of a myocardial infarction (MI). He recently presented with angina symptoms and had a cardiac catheterization which did not reveal any concerning areas of stenosis. He has not been taking his medications. Physical examination shows his pulse is 88 beats/minute, respiration rate is 16 breaths/minute, and blood pressure is 130/80 mmHg. His lungs are clear, heart sounds are normal, and there is no lower-extremity edema. Which of the following is the best treatment to manage the chest pain? A. Angiotensin-converting-enzyme (ACE) inhibitor B. Baby aspirin C. Beta blocker D. Calcium channel blocker E. Statin

The answer is C. From IM 02. This patient has stable angina and should be treated with a beta blocker. The beta blocker will reduce angina by slowing his heart rate and decreasing his blood pressure, thus decreasing myocardial oxygen consumption.

A 65-year-old female is brought to the emergency department by the paramedics after a syncopal event. She reports a prodrome of lightheadedness and diaphoresis while straining to have a bowel movement. Her husband says that she lost consciousness for fewer than five minutes in the bathroom. Her medical history is significant for diabetes mellitus type 2 for the past 10 years. She takes metformin, aspirin, and furosemide daily. She is currently awake and oriented to person, place, and time. Her vital signs are normal, including orthostatic vitals, and she was able to stand for the orthostatic vitals without symptoms. Physical examination and routine blood tests are normal. Her electrocardiogram (ECG) is normal. Which of the following is the most likely etiology of syncope in this patient? A. Hypoglycemia B. Medication-induced bradycardia C. Neurocardiogenic syncope D. Orthostatic syncope E. Tachyarrhythmia

The answer is C. From IM 03.

A 66-year-old male is admitted to the hospital for treatment of new-onset atrial fibrillation with a rapid ventricular rate. The troponin, complete blood count (CBC), and basic metabolic profile are within normal limits. Which additional laboratory test is indicated as part of the initial workup for new-onset atrial fibrillation? A. Creatine kinase-MB (CK-MB) B. Prothrombin time and international normalized ratio (ProTime INR) C. Thyroid stimulating hormone (TSH) D. Urinalysis E. Vitamin B12

The answer is C. From IM 03.

A 42-year-old female comes to the clinic for a follow-up of newly diagnosed type 2 diabetes mellitus. She is asymptomatic and has a normal physical exam. Her blood pressure is 142/76 mmHg, which is consistent with previous blood pressure measurements at home. Results of lab work show 1+ protein on her urinalysis and a normal-basic metabolic panel. What is the most appropriate treatment for the patient? A. Amlodipine B. Hydrochlorothiazide C. Lisinopril D. Metoprolol E. No further treatment

The answer is C. From IM 06.

An 82-year-old female with diabetes, hypertension, hyperlipidemia, and a stroke three years ago with residual dysarthria, is admitted to the intensive care unit (ICU) after her daughter was unable to wake her up this morning. On exam, she is afebrile, and her neurological exam is not focal. A head computed tomography (CT) scan did not reveal any acute pathology. Labs are as follows: white blood count (WBC) 12.4 cells x 10 3 /μL sodium 123mEq/L, potassium 3.4mEq/L, chloride 107 mEq/L, bicarbonate 22 mEq/L, blood urea nitrogen (BUN) 42 mg/dL, creatinine (Cr) 2.3 mg/dL, glucose 840 mg/dL. A serum osmolality is 410 mosm/kg. In addition to insulin, which of the following interventions is most likely to improve her outcome? A. Antibiotics B. Dialysis C. Intravenous (IV) fluids D. Potassium supplementation E. Sodium bicarbonate

The answer is C. From IM 07.

A 46-year-old female with diabetes and obesity presents to the office for a routine follow-up. She is currently asymptomatic and takes aspirin and metformin. She would like to lose weight. On physical examination, her pulse is 76 beats/minute, blood pressure is 138/72 mmHg, and body mass index (BMI) is 42 kg/m2. She asks if there is anything she could be doing to improve her health. Which of the following is the most appropriate counseling statement for this patient? A. "I think you should try orlistat to lose weight." B. "We need to refer you to the bariatric surgeon given your diabetes and high BMI." C. "We will work together on trying to achieve a 5-10% loss in weight over the next six months." D. "You should exercise more in order to lose weight." E. "You should attempt to lose enough weight to achieve a normal BMI."

The answer is C. From IM 08.

A 38-year-old male with alcohol use disorder is admitted to the hospital medicine service. He presented to the emergency department with shaking that began earlier today. His last drink was 16 hours ago. He has been drinking a pint of vodka daily for the past 10 years. On physical exam, he is disheveled and unkempt. His vitals reveal temperature is 37.2 C (99 F), pulse is 105 beats/minute, respiratory rate is 16 breaths/minute, and blood pressure is 165/95 mmHg. He has a fine tremor with his arms outstretched without asterixis. Except for the tachycardia, his cardiac exam is unremarkable. He has an unsteady gait and some visual hallucinations, but is fully oriented. He denies any other ingestions or any daily medications. Which of the following should your treatment plan include? A. A beer with each meal B. Cobalamin C. Diazepam D. Folate E. Phenobarbital

The answer is C. From IM 09.

A 55-year-old male is seen in the emergency department for acute abdominal pain. He is lying motionless when you interview him. On exam, there are minimal bowel sounds. He has diffuse abdominal tenderness. His abdomen is hard when you palpate it, and he is unable to relax his abdominal muscles after you warm up your hands, distract him, and palpate gently. When you accidentally bump the stretcher, he cries out in pain. What type of pain is this patient experiencing? A. Ischemic pain from celiac artery occlusion B. Obstruction of a hollow viscera C. Parietal pain from inflammation of the parietal peritoneum D. Referred abdominal pain from an inferiorly located pneumonia E. Somatic body-wall pain from an abdominal wall muscle injury

The answer is C. From IM 09.

Patients with chronic hepatitis C virus (HCV) infection are at risk of fulminant hepatic failure, if co-infection with hepatitis A virus (HAV) occurs. Which if the following factors poses the greatest risk for infection with HAV? A. Blood transfusion B. Consumption of undercooked shellfish C. Foreign travel D. Illicit drug use E. Working at a daycare

The answer is C. From IM 11.

A 24-year-old female presents to the emergency department with 18 hours of lower abdominal pain, nausea, vomiting, and diarrhea. She is afebrile and has normal vital signs except for tachycardia with a pulse at 105 beats/minute. Her abdomen is soft and non-distended with hypoactive bowel sounds. There is rebound tenderness in the left-lower quadrant. The abdominal exam finding of rebound tenderness has +LR = 2 and -LR = 0.4 for peritonitis. What is the best interpretation of this finding? A. Absence of rebound tenderness strongly argues against peritonitis B. Absence of rebound tenderness strongly argues against peritonitis, but presence of it does not affect the probability of peritonitis C. Neither the presence nor the absence of rebound tenderness is very helpful in diagnosing peritonitis D. Presence of rebound tenderness strongly suggests peritonitis E. Presence of rebound tenderness strongly suggests peritonitis, but absence of it does not affect the probability of peritonitis

The answer is C. From IM 12.

A 51-year-old male is admitted to the hospital for acute, uncomplicated diverticulitis. Medical history is significant for asthma, gastroesophageal reflux disease (GERD), and hypertension. He is treated with intravenous ciprofloxacin and metronidazole, intravenous fluids, and pain medication. After two days, his pain is improved and he is tolerating food. Which of the following discharge plans is most appropriate for him? A. Continue oral antibiotics for 21 days B. Prescribe subcutaneous enoxaparin for 28 days C. Refer to gastroenterology for a colonoscopy D. Refer to a nutritionist to initiate a gluten-free diet E. Refer to surgery for resection of the affected bowel

The answer is C. From IM 12.

A 63-year-old female with a past history of hypertension, chronic obstructive pulmonary disease (COPD), type 2 diabetes mellitus, and hyperlipidemia is admitted to the hospital with Clostridium difficile (C. difficile) colitis. Of the following options, which is the best choice for deep vein thrombosis (DVT) prophylaxis for her? A. Ambulation B. Aspirin, 81 mg by mouth (PO) daily C. Enoxaparin, 40 mg subcutaneous daily D. Heparin intravenous (IV) continuous infusion, titrated for a goal-activated partial thromboplastin time (aPTT) of 50 to 70 seconds E. Intermittent pneumatic compression (IPC) device

The answer is C. From IM 12.

A 35-year-old female presents with urgency, frequency, and pain when she urinates. Other associated symptoms include left-lower-back pain, fever, chills, nausea, and vomiting. She has no medical problems and does not take any medications. On physical examination her vital signs show her temperature is 38.9 C (102 F), blood pressure is 90/55 mmHg, pulse is 120 beats/minute, respiratory rate is 18 breaths/minute, and oxygen saturation is 98% on ambient air. She has left flank tenderness and lower abdominal tenderness. Laboratory studies show an elevated white blood cell count, but her basic metabolic panel is unremarkable. Urinalysis reveals pyuria with white blood cell casts. Blood and urine cultures are sent. In addition to intravenous fluids and hospital admission, which of the following is the most appropriate next step? A. Azithromycin B. Cefpodoxime C. Ceftriaxone D. Moxifloxacin E. Vancomycin and piperacillin/tazobactam

The answer is C. From IM 14.

A 50-year-old male with a history of hypertension presents with urgency, frequency, and pain when he urinates. He reports no fever, but has chills, lower abdominal pain, cloudy urine, and nausea. He notes he had a similar presentation about a year ago that resolved with antibiotics. He states that he has a weak urinary stream and a sensation that his bladder is full even after urinating. His current medications are acetaminophen and amlodipine. On physical examination his vital signs are normal, but his examination is limited due to severe suprapubic tenderness. Laboratory studies show an elevated white blood cell count, normal basic metabolic panel, and negative gonorrhea and chlamydia testing. His urinalysis is consistent with a urinary tract infection (UTI). In addition to antibiotic treatment, which of the following is the most appropriate next step? A. Cystoscopy B. No further management C. Post-void residual bladder volume D. Repeat the urinalysis and urine culture E. Transurethral resection of the prostate

The answer is C. From IM 14.

A 32-year-old postpartum female comes to see you in the clinic. She has concerns about "splotchy" dark spots above her lip that do not bother her aside from their "ugly" appearance. She reports no acne or other skin conditions such as rosacea. On exam, you note hyperpigmented macules on the upper lip. What is her diagnosis? A. Café au lait spots B. Fixed-drug eruption C. Melasma D. Postinflammatory hyperpigmentation E. Solar lentigo

The answer is C. From IM 17.

A 77-year-old female is brought to the office by her daughter with complaints of fatigue, poor appetite, and forgetfulness that have been getting progressively worse over the past three months. Conversing with the patient is difficult, as she only provides one-word answers to your questions; her daughter provides most of the history. The patient has had weight loss and constipation for which she has been taking glycerin suppositories. She has had more difficulty bathing and decreased motivation to practice good hygiene. She lives alone, but her daughter is nearby and has been visiting her with increasing frequency over the past month. On physical examination, she is afebrile. Her pulse is 120 beats/minute. Her blood pressure is 140/90 mmHg. She appears frail and disheveled. A mini-mental state examination (MMSE) is performed, and her score is 7. What would be the ​next best step in management? A. No further lab testing needed. Start donepezil. B. Order a serum folate level C. Order a thyroid stimulating hormone (TSH) level D. Order a transthoracic echocardiogram E. Order an erythrocyte sedimentation rate (ESR)

The answer is C. From IM 18.

An 80-year old male with non-obstructive coronary artery disease, diabetes mellitus type 2, and hypertension presents after a fall he had at home about five hours ago. He felt well this morning. After a light breakfast, he read the newspaper at the kitchen table. When he stood up, he felt lightheaded and fell to the floor. His daughter, who lives with him, heard the thud and found him on the floor. He was orientated to person, place, and time when found by his daughter. He denied having any urinary or stool incontinence. He also has not had fevers, palpitations, abdominal discomfort, hematuria, or dysuria. As he was complaining of left hip pain, she brought him into the emergency room for evaluation. He has been prescribed aspirin, metoprolol, enalapril, amlodipine, and metformin, all of which he took this morning. The enalapril was recently added to his regimen. On exam, vitals were as follows: temperature is 37.6 C (99.7 F), pulse is 60 beats/minute, and blood pressure is 110/70 mmHg. You find a pleasant male who appears his stated age and is in no distress. His exam, including the neurological exam, is unremarkable except for ecchymoses over the left hip. He scores a 29 on the mini-mental state examination (MMSE). He is unable to complete a "Get Up and Go" test due to pain in his left hip. Initial tests show a finger stick glucose of 220 mg/dL, sodium 135 mmol/L, creatinine 1.0 mg/dL, white blood cell count 9,000 cells/mL, hemoglobin 14 g/dL, and troponin I 0.01 ng/mL. A urinalysis showed 4+ bacteria with no leukocyte esterase or nitrites. An electrocardiogram (ECG) showed a normal sinus rhythm with significant Q waves in leads II, III, and avF. A non-contrast computed tomography (CT) of the head shows no structural lesions. A pelvic radiograph and magnetic resonance imaging (MRI) of the pelvis both show no evidence of fracture. What is the most likely contributor to his fall? A. Acute coronary syndrome B. Hypoglycemia C. Medications D. Seizure disorder E. Urinary tract infection

The answer is C. From IM 18.

An 80-year-old female presents to the office with urinary incontinence. She has a history of congestive heart failure (CHF) and severe osteoarthritis of her knees. She takes furosemide, but states that she often misses doses because it makes her urinate more frequently. She is embarrassed when she has urinary incontinence while out of the house. She has no dysuria or difficulty controlling her bowel movements. She can feel the urge to urinate, and had rare difficulty with incontinence prior to starting diuretics. Her exam is significant for clear lungs, severe crepitus of both knees, and a normal neurologic exam. Her "Timed Up and Go" test is 30 seconds. What is the next step in the management of her urinary incontinence? A. Discontinue diuretics B. Initiate tamsulosin C. Modify the timing of her diuretic therapy D. Order a computed tomography (CT) of her abdomen and pelvis E. Place an indwelling urinary catheter

The answer is C. From IM 18.

A female with a severe headache undergoes a lumbar puncture, and red blood cells (RBCs) are found in the cerebrospinal fluid (CSF) specimen. Which of the following additional findings would best support the hypothesis that a traumatic tap caused the presence of RBCs in the CSF specimen? A. CSF glucose lower than serum glucose B. Elevated protein in the CSF C. Fewer RBCs in each subsequent tube of CSF fluid analyzed D. Positive Gram stain of the CSF E. Presence of xanthochromia in the CSF fluid

The answer is C. From IM 24.

A patient is admitted to the hospital for suspected bacterial meningitis. A lumbar puncture is performed, and the results are pending. While awaiting the cerebrospinal fluid (CSF) results, which of the following types of isolation would be the most appropriate for this patient? A. Airborne isolation B. Contact isolation C. Droplet precautions D. No isolation E. Standard precautions

The answer is C. From IM 24.

An 80-year-old Spanish-speaking female who lives alone and has no medical information available was found down after an unknown period of time. She is confused and unable to provide a reliable history, even with a Spanish interpreter present throughout your interview. Her exam is notable for normal vital signs and tenderness over the right pubic rami. Hip and pelvis images reveal two pubic rami fractures, but no significant hemorrhage. Laboratory workup is significant for blood urea nitrogen (BUN) of 30 mg/dL and creatinine (Cr) of 1.5 mg/dL. While the Spanish interpreter is still available, your resident asks you to use the Confusion Assessment Method to determine whether delirium is the cause of the patient's confusion. Which of the following is the best method to assess for inattention? A. Count backwards from 100 by increments of seven B. Describe name, location, and date C. Recite the days of the week forward and backward D. Remember three objects after two minutes E. Spell "world" forwards and backwards

The answer is C. From IM 25.

A 55-year-old male with a 40-pack-per-year smoking history presents with a several month history of a 15-pound unintentional weight loss, and a worsening cough. He reports polyuria and polydipsia. Physical exam is significant for a wheezing in the lower lung fields bilaterally. His calcium level is 12.5 mg/dL. A chest x-ray is significant for a 2 cm lung nodule next to the left bronchus and prominent hilar lymphadenopathy. What is the next-best test that should be ordered to work up this patient's hypercalcemia? A. 1,25(OH)2 Vitamin D B. Cortisol C. Parathyroid hormone (PTH) D. Parathyroid hormone-related peptide (PTH-rp) E. Thyroid stimulating hormone (TSH)

The answer is C. From IM 27.

An 85-year-old female with widely metastatic breast cancer presents to the emergency department with confusion, fatigue, constipation, and global weakness. Her daughter reports that she has been drinking over three liters a day of water and juice. She has not had a bowel movement in five days. Her labs are significant for a calcium of 15mg/dL. An electrocardiogram (ECG) is ordered. What finding is the hypercalcemia most likely to cause on her ECG? A. Atrial fibrillation B. PR segment depression C. Shortened QT interval D. Sinus tachycardia E. T-wave inversion

The answer is C. From IM 27.

You are seeing a delightful 62-year-old female in the primary care clinic for follow-up of pulmonary function tests (PFTs). She has long been your patient, although you have never been able to get her to quit smoking. She has had no hospitalizations since her children were born. Over the past three months she has felt dyspneic with exertion. Her complete blood count (CBC), basic metabolic panel, and stress echo were normal. Her PFTs showed a post-bronchodilator FEV1/FVC of 65% and FEV1 of 41% predicted. Based on this data, what additional information do you need to guide pharmacologic therapy? A. Computed tomography (CT) of her chest B. Folate level C. History of breathlessness with getting dressed D. History of headaches E. Troponin T measurement

The answer is C. From IM 28.

A 65-year-old female presents to the emergency department with one month of worsening dyspnea. She reports no fever or cough. Her past medical history is significant for diabetes. She has a 20-pack-per-year smoking history. Vitals show her temperature is 37 C (98.6 F), pulse is 100 beats/minute, respiratory rate is 24 breaths/minute, blood pressure is 120/80 mmHg, and oxygen saturation of 96% on two liters of oxygen by nasal cannula. A chest x-ray shows a large left-sided pleural effusion. A thoracentesis is performed. Serum and pleural fluid results are shown below: Serum protein: 6.0 g/dL Serum lactate dehydrogenase (LDH): 400 U/L Pleural fluid studies: Pleural fluid LDH: 1500 U/L Pleural fluid total protein: 5.5 g/l Pleural fluid glucose: 66 mg/DL Pleural fluid red blood cells (RBCs): 6.5 cells x 106 /µL Pleural fluid white blood cell count (WBC): 4.0 cells x 103/L; 80% lymphocytes. Gram stain shows no organisms. Which of the following is the most appropriate next step in management? A. Consult a thoracic surgeon to place a chest tube B. Refer for pleural biopsy C. Send pleural fluid for cytology D. Treat with ceftriaxone and azithromycin E. Treat with vancomycin and piperacillin-tazobactam

The answer is C. From IM 29.

You are called to the bedside to evaluate a 45-year-old male with acute dyspnea. He has been hospitalized for treatment of an exacerbation of inflammatory bowel disease for five days. He had no hypoxia or tachypnea on admission. He has no cough. Current medications include methylprednisolone and morphine. Vital signs are significant where temperature is 37.5 C (99.5 F), pulse is 110 beats/minute, respiratory rate is 30 breaths/minute, blood pressure is 120/80 mmHg, and oxygen saturation is 92% on 2 liters. He appears uncomfortable with mild use of accessory muscles of respiration. Lungs are clear to auscultation bilaterally. A chest x-ray shows no infiltrate, effusion, or pulmonary edema. His basic metabolic profile is normal. What is the most-appropriate next step in management? A. Albuterol inhaler B. Blood and urine cultures C. Computed tomography pulmonary angiogram (CTPA) D. Pulmonary function testing E. Vancomycin and cefepime

The answer is C. From IM 29.

A 26-year-old male presents to the emergency department with a three-day history of chest pain. The pain is described as sharp, left-sided, and exacerbated by breathing. He reports having no exertional dyspnea. He is previously healthy. Vital signs are normal. He has no tenderness with palpation of the chest wall over the area where he has pain. There is 1+ pitting edema in the right leg. The remainder of the examination is normal. What is the next most-appropriate test to determine the probability of thromboembolism in this patient? A. Chest radiograph B. Computed tomography angiogram (CTA) of the chest C. High sensitivity D-dimer D. Lower-extremity (LE) duplex ultrasound E. Ventilation-perfusion (V/Q) scan

The answer is C. From IM 30.

A 32-year-old female with no known medical history presents with two weeks of pain in her right knee, fever, and an associated effusion. You evaluate her and plan to perform an arthrocentesis to further elucidate the cause of the effusion. You review the indications for the arthrocentesis, but the patient asks you if there are any risks. Which of the following, if any, is a risk of an arthrocentesis? A. Dry tap B. Introduction of calcium pyrophosphate crystals into the joint C. Introduction of infection into the joint D. None E. Radiation exposure from ultrasound guidance

The answer is C. From IM 31.

A 50-year-old female comes to the office with joint pain. She has pain, swelling, and warmth in her bilateral metacarpophalangeal (MCP) and proximal interphalangeal (PIP) joints, wrists, and ankles consistent with chronic inflammatory polyarthritis. She has no skin or oral concerns. Which of the following is the most common cause of this patient's clinical presentation? A. Dermatomyositis B. Gonococcal arthritis C. Rheumatoid arthritis D. Systemic lupus erythematosus E. Systemic vasculitis

The answer is C. From IM 32.

A 50-year-old male presents with progressive bilateral knee and ankle pain. You are concerned about a possible inflammatory arthritis. Which of the following key findings on history is most consistent with an inflammatory etiology of this patient's pain? A. Monoarthritis with weakness B. Morning stiffness lasting less than one hour C. Polyarthritis with joint warmth D. Previous trauma to affected joint(s) E. Recent overuse of affected joint(s)

The answer is C. From IM 32.

A 54-year-old male is being evaluated for two months of left knee pain. Over the same time period, he has also noticed a "creaking" sensation when walking down stairs and stiffness in the morning that lasts about 20 minutes. He is otherwise feeling well. On exam, there is visible bony enlargement and some mild bony tenderness, but no warmth or effusion. Which of the following is the most-likely diagnosis? A. Anterior cruciate ligament (ACL) tear B. Gout C. Osteoarthritis D. Rheumatoid arthritis E. Septic arthritis

The answer is C. From IM 32.

A 33-year-old male presents to the emergency department for persistent fever 38.3 C to 38.9 C (101 F to 102 F), night sweats, and fatigue for the past four weeks. This is the second time he has come to the emergency department for these concerns. He does not have insurance and does not have a primary care provider. During his first emergency department visit one week ago, his evaluation included a detailed but non-revealing history and physical examination, normal labs (including complete blood count, complete metabolic panel, erythrocyte sedimentation rate, c-reactive protein, human immunodeficiency virus (HIV) and RNA polymerase chain reaction (PCR), urinalysis, urine culture, blood cultures, lactate dehydrogenase, antinuclear antibodies), negative chest x-ray, and a negative computed tomography (CT) of the chest and abdomen. Which of the following would be the most-appropriate next step? A. Perform a bone marrow biopsy B. Perform a tagged white cell scan C. Perform interferon gamma release assay for tuberculosis D. Repeat blood culture and start empiric antibiotics E. Start therapeutic trial of corticosteroids

The answer is C. From IM 35.

A 32-year-old female comes to the emergency department with increasing abdominal distension and shortness of breath over the past week. She reports no fever or chills but has had decreased intake due to early satiety. Medical history is positive for autoimmune hepatitis. On exam, she is mildly tachypneic but her lungs are clear to auscultation bilaterally. Cardiac exam is normal. Her abdominal exam is significant for marked distension and a positive fluid wave, without tenderness on palpation. You perform a paracentesis and remove five liters of fluid. The fluid is clear with 15 white blood cells (WBCs), 90% neutrophils, 20% leukocytes. The paracentesis fluid albumin is 0.2 g/dL, lactate dehydrogenase (LDH) is 50 U/L, and protein is 1.2 g/dL. The serum albumin is 1.9 g/dL, LDH is 170 U/L, and total protein 5.9 g/dL. Which of the following is the most likely cause of the patient's ascites? A. Congestive heart failure B. Metastatic lung cancer C. Portal hypertension D. Secondary peritonitis E. Spontaneous bacterial peritonitis

The answer is C. From IM 36.

A 37-year-old male with a history of alcoholic cirrhosis is brought in to the emergency department by his husband with confusion and lethargy. He was otherwise doing well until yesterday when he slept most of the day and refused his medications. This morning, he was difficult to wake up, and when he did wake up, he was very confused and somewhat combative. His husband reports that up until yesterday, he was adherent to his medications and has not drunk alcohol in four months. The husband says this has never happened to the patient before. On exam, the patient's temperature is 38.5 C (101.3 F), pulse is 98 beats/minute, respiratory rate is 12 breaths/minute, and his blood pressure is 94/64 mmHg. He is somnolent and only arousable to sternal rub. When he does wake up, he can only tell you his name. He is notably jaundiced. His heart and lung exams are normal. His abdomen is distended with a positive fluid wave and flank dullness. He winces as you palpate his abdomen without a clear localization. What is the most likely cause of the patient's altered mental status? A. Alcohol intoxication B. Hepatic encephalopathy C. Spontaneous bacterial peritonitis D. Urinary tract infection E. Variceal bleeding

The answer is C. From IM 36.

A 48-year-old male with known alcohol abuse history comes to the hospital with abdominal discomfort and early satiety. He has no fevers or chills. He describes the discomfort as generalized. Eating makes it worse, while belching can make it better. The pain does not radiate, and he rates it typically as 3/10. He says his pants no longer fit him well. His abdomen has been getting bigger and his legs are more swollen--both of which he attributes to drinking more alcohol recently. His only medication is an antihypertensive. His last drink was earlier this morning. What physical examination finding would be most specific to identify the cause of this abdominal swelling? A. Caput medusae B. Flank dullness C. Fluid wave D. Lower extremity edema E. Shifting dullness

The answer is C. From IM 36.

A 67-year-old male is admitted to the hospital with a non-ST elevation myocardial infarction. He took no medications prior to admission. During his hospital stay, he is noted to have hypertension. Echocardiogram shows left ventricular hypertrophy, an ejection fraction of 60%, and normal wall motion and valvular function. Which of the following is the best antihypertensive agent to start in this patient? A. Hydralazine B. Isosorbide mononitrate C. Lisinopril D. Metoprolol

The answer is D. From IM 01.

A 70-year-old female presents to the emergency department with moderately severe chest pain. She describes the pain as a vague, deep discomfort and notes associated nausea, diaphoresis, and dyspnea. The pain has no clear aggravating or alleviating factors. Her pain began early on the morning of presentation, waking her from sleep, and has persisted for two hours. Her exam, including vital signs, is normal. What is the most important initial diagnostic test? A. Chest x-ray B. Complete blood count C. Computed tomography (CT) angiography of the chest D. Electrocardiogram (ECG) E. Troponin

The answer is D. From IM 01.

A 46-year-old female presents to her primary care provider to discuss her risk of cardiovascular disease. She notes that her mother had a heart attack at age 66. She denies any recent symptoms of chest pain or shortness of breath. She has a history of hypertension, hyperlipidemia, and obesity. She is a never smoker and drinks a glass of wine daily. Current medications are 10mg amlodipine daily, and 40mg atorvastatin daily. Her pulse is 78 beats/minute, respiration rate is 16 breaths/minute, and blood pressure is 154/80 mmHg. Her lungs are clear, heart sounds are normal, and there is no lower-extremity edema. Her electrocardiogram (ECG) is normal. A recent fasting lipid profile obtained 2 weeks prior to this visit shows a total cholesterol of 173mg/dL, an HDL of 45mg/dL, and an LDL of 105mg/dL. Which of the following is the most appropriate management to lower her risk of cardiovascular disease? A. Limit alcohol intake B. Limit polyunsaturated fats C. Start a beta blocker D. Start a thiazide diuretic E. Take an aspirin daily

The answer is D. From IM 02. This patient should have her blood pressure medication titrated upward to optimize her risk reduction. Thiazide diuretics are first-line antihypertensive agents and should be added to her regimen to achieve goal blood pressure value < 130/80.

A 50-year-old male is brought to the hospital by paramedics after a witnessed syncopal event. The event occurred at a local baseball game where he was a spectator. He was evaluated by paramedics at the scene within three minutes of the event and arrived at your hospital within 15 minutes. An intravenous (IV) line was started, but no fluid was given. The patient states that he stood up to get some lunch and suddenly became very lightheaded. He collapsed and does not recall anything until he awoke with many bystanders attending to him. He was aware of his surroundings immediately upon regaining consciousness and was not confused. Bystanders state he had a 30-second period of loss of consciousness and did not hit his head when he collapsed. His vital signs are as follows: temperature is 36.7 C (98 F), pulse is 90 beats/minute, respiratory rate is 12 breaths/minute, and blood pressure is 110/65 mmHg. What is the next best step to determine the cause of the patient's syncopal episode? A. Assessment for pain with passive ankle dorsiflexion (Homan's sign) B. Auscultation of the carotid arteries C. Cranial nerve exam D. Orthostatic blood pressure measurement E. Pulmonary exam

The answer is D. From IM 03.

A 68-year-old female is admitted to the hospital because of increasing dyspnea and orthopnea over the past two weeks. Her medical history is significant for ischemic cardiomyopathy with an ejection fraction of 40% and low back pain for one month. She reports no chest pain, palpitations, fever, and cough. She has been adherent to a low-sodium diet. She does not use alcohol or tobacco. Daily medications are carvedilol, lisinopril, amlodipine, atorvastatin, aspirin, and ibuprofen. Which medication would most likely cause her worsening symptoms? A. Amlodipine B. Atorvastatin C. Carvedilol D. Ibuprofen E. Lisinopril

The answer is D. From IM 04.

A 31-year-old male with previously diagnosed hypertension comes in for a follow-up visit. He currently takes 10 mg of amlodipine daily, 40 mg of enalapril daily, and 100 mg of atenolol daily. His pulse is 65 beats/minute. His blood pressure is 190/100 mmHg in his right arm and 188/100 mmHg in his left leg. He reports adherence to all of his medications, and he took all of them this morning. He reports no headache, vision changes, chest pain, palpitations, or shortness of breath. He does not take any over-the-counter medications. He reports no tobacco, alcohol, or drug use. His physical exam is unremarkable. His basic metabolic profile reveals: Sodium 139 Potassium 3.1 Chloride 103 Bicarbonate 21 BUN 10 Creatinine 1.1 Glucose 87 Which of the following is the most likely cause for the man's hypertension? A. Aortic coarctation B. Essential hypertension C. Pheochromocytoma D. Primary hyperaldosteronism E. Renal artery stenosis

The answer is D. From IM 06.

A 53-year-old male comes in for his yearly checkup. He has no past medical history and takes no medications. He rushes in five minutes after his appointment time because his bus ran late, and he had to run one block to get to the office. He is sweaty and still breathing heavy. His blood pressure taken by the medical assistant is 157/85 mmHg in his right arm. What is the next best step? A. Check a basic metabolic panel (BMP) for possible hypertensive renal disease B. Prescribe ambulatory blood pressure monitoring C. Recheck his blood pressure in the other arm D. Repeat his blood pressure after he has been seated quietly for five minutes E. Start him on 5 mg of amlodipine daily

The answer is D. From IM 06.

A 28-year-old Caucasian female presents to the clinic for her annual exam. She has no complaints. On exam, her pulse is 84 beats/minute, her blood pressure is 116/70 mmHg, and her body mass index (BMI) is 28 kg/m2. Which of the following would prompt screening for diabetes? A. A hemoglobin A1c (HbA1c) of 5 at last year's exam B. A mother recently diagnosed with hypothyroidism C. A paternal aunt with diabetes D. A history of gestational diabetes with her last pregnancy, seven years ago. E. Frequent consumption of refined carbohydrates

The answer is D. From IM 07.

A 52-year-old male with hypertension and hyperlipidemia comes to the clinic for a routine follow-up. He has no symptoms. His mother was recently diagnosed with diabetes, and he wants to know if he is also diabetic. He quit smoking two years ago after being admitted to the hospital with chest pain. He has gained 30 pounds since that time, and his body mass index (BMI) is currently 30 kg/m2. His last meal was breakfast two hours ago, which consisted of a bagel and orange juice. Which of the following studies is most likely to support a diagnosis of diabetes? A. Blood glucose level in the clinic of 160 mg/dL B. Blood glucose level of 190 mg/dL two hours after a 75 g glucose load C. Decreased sensation in feet by monofilament test D. Hemoglobin A1c (HbA1c) of 6.8% E. Urinalysis with glucosuria

The answer is D. From IM 07.

A 52-year-old female comes to the clinic for a health maintenance visit. Past medical history is significant for hypertension, hyperlipidemia, and diabetes. Her body mass index (BMI) from three months ago was 31.5 kg/m2. For the past three months, she has restricted calories and increased her aerobic activity. Current medications are lisinopril, aspirin, atorvastatin, and metformin. Her pulse is 76 beats/min, blood pressure is 138/76 mmHg, and BMI is 31 kg/m2. The physical exam is normal. Her hemoglobin A1c (HgbA1c) is 7.6%. The patient is frustrated, as her weight has not improved significantly. Involvement of which of the following individuals would be most appropriate in the care of this patient? A. Cardiologist B. Endocrinologist C. Physical therapist D. Registered nutritionist E. Social worker

The answer is D. From IM 08.

A 52-year-old female with hypertension, diabetes, and obesity presents to the office for a routine follow-up. She is currently asymptomatic and on lisinopril, aspirin, simvastatin, and metformin. On physical examination, her pulse is 76 beats/minute, blood pressure is 128/72 mmHg, and body mass index (BMI) is 32 kg/m2. She is due for an annual examination of her feet. Which of the following is the most appropriate component of the foot exam for this patient? A. Assessment of Achilles tendon thickness B. Assessment of femoral pulses C. Assessment of patellar reflex D. Inspection between toes for evidence of skin breakdown E. Use of monofilament to assess temperature sensation

The answer is D. From IM 08.

A 51-year-old female with a history of rheumatoid arthritis and coronary artery disease is admitted to the hospital with acute liver failure. During the admission process, the team calls her pharmacy and primary care physician and confirms the doses and route of administration and her adherence to her prescription medications. What additional information should be obtained in the medication reconciliation process? A. Allergies to medications B. Cost of prescriptions C. List of all medications taken in the last five years D. List of over-the-counter medications taken E. Number of refills remaining on prescriptions

The answer is D. From IM 10.

A 21-year-old male with history of hepatitis C, cholelithiasis, sickle cell disease, and alcohol use disorder presents to the hospital with one day of severe abdominal and chest pain, worse in the right-upper quadrant. A fractionated bilirubin reveals the following: Total bilirubin: 5.6 Indirect bilirubin: 4.6 Direct bilirubin: 1.0 What is the most likely cause of the bilirubin pattern in this patient? A. Alcoholic hepatitis B. Choledocholithiasis C. Non-alcoholic steatohepatitis D. Sickle cell crisis E. Viral hepatitis

The answer is D. From IM 11.

A 40-year-old female comes to your office after an emergency department visit for abdominal pain, which has subsequently resolved. However, an ultrasound at that time suggested non-alcoholic fatty liver disease. She is worried about developing cirrhosis. Which of the risk factors is most likely associated with the progression of non-alcoholic fatty liver disease (NAFLD) to non-alcoholic steatohepatitis (NASH), and eventually liver fibrosis (cirrhosis)? A. High amounts of coffee consumption B. Low body mass index (BMI) C. Low low-density lipoprotein (LDL) D. Type 2 diabetes mellitus E. Younger age

The answer is D. From IM 11.

A 49-year-old female is evaluated for fatigue, pruritus, dry eyes, and dry mouth. Her symptoms have been present for about four months and have progressively worsened. She denies any fever, chills, or weight changes. She has hyperlipidemia for which she takes atorvastatin. She has no other medical problems. Her vital signs are normal, and body mass index (BMI) is 24 kg/m2. Her physical exam is notable for excoriations on her arms, legs, chest, and back; otherwise unremarkable. Her alanine transaminase (ALT) is 76 U/L, aspartate aminotransferase (AST): 55 U/L, alkaline phosphatase (ALP): 356 U/L, total bilirubin: 1.3 mg/dL, and direct bilirubin: 0.7 mg/dL Her right-upper-quadrant ultrasound is normal.Which of the following is the most likely diagnosis? A. Cirrhosis B. Crigler-Najjar syndrome C. Gilbert syndrome D. Primary biliary cholangitis (PBC) E. Wilson disease

The answer is D. From IM 11.

Which of the following factors carries the highest risk for contracting hepatitis C? A. Birth between 1945 and 1965 B. Blood transfusion C. Consumption of more than three alcohol-containing drinks per day D. Injection drug use E. Sexual contact with an injection drug user

The answer is D. From IM 11.

Mrs. Grant is a 72-year-old female with New York Heart Association (NYHA) grade 3 heart failure with a left ventricular ejection fraction of 30%, severe chronic obstructive pulmonary disease (COPD) with ongoing tobacco use, and a recent diagnosis of dementia. During her office visit, her daughter asks if Mrs. Grant should get a colonoscopy. Her last colonoscopy, which was normal, was done when she was 62 years old. She does not have any gastrointestinal symptoms. On examination, pulse is 92 beats/minute, blood pressure is 106/72 mmHg, and body mass index (BMI) is 20.4 kg/m2. She is barrel-chested, has a prolonged expiratory phase, scattered wheezes, and bibasilar crackles. Cardiac exam is regular with a grade 2/6 systolic murmur at the apex. She has 1+ bilateral pedal edema. What approach would you recommend regarding a screening colonoscopy? A. Discourage her, because the test would not be covered by Mrs. Grant's insurance B. Insist that Mrs. Grant have the colonoscopy as soon as possible C. Perform an alternative screening test for colon cancer, such as a fecal immunochemical test (FIT) D. Recommend against colonoscopy due to comorbid conditions and life expectancy E. Schedule a colonoscopy once Mrs. Grant's ejection fraction improves and she quits smoking

The answer is D. From IM 13.

Ms. May is a 28-year-old female who comes to see you because her 58-year-old mother was recently diagnosed with breast cancer. Ms. May has no medical issues and takes no medications. A detailed family history reveals that her maternal great-aunt died of breast cancer at 65 years old. She has no relatives with ovarian cancer. Her breast examination is normal. Based on U.S. Preventive Services Task Force (USPSTF) guidelines, what would you recommend for breast cancer screening for Ms. May? A. Mammography now and then every year after the age of 40 B. Monthly breast self-examination with mammography beginning at the age of 40 C. Routine mammography every two years beginning at the age of 50 D. Shared decision-making discussion regarding initiation of mammography E. Yearly mammography at the age of 60

The answer is D. From IM 13.

A 25-year-old female presents for advice on contraception methods. She has a history of migraine with aura, but states she has not had a migraine in six months. She takes non-steroidal antiinflammatory drugs as needed for the migraines. She is working at her "dream job", although she often has to work 12- to 14-hour days. She lives with her boyfriend, and they will be getting married in two months. She would like to start having children in about two to three years. She has normal monthly menstrual cycles that last about four to five days. Her last menstrual period was two days ago. She states no alcohol or illicit drug use, but admits that she often smokes 5-10 cigarettes a week and is trying to quit. Physical examination and vital signs are unremarkable. She is requesting the most-effective contraception method against pregnancy that can accommodate her busy work schedule. She is concerned about weight gain. What would be the most appropriate contraception for this patient? A. Condoms B. Depot medroxyprogesterone acetate C. Estrogen/progestin contraceptive pills D. Levonorgestrel intrauterine device E. Post-coital contraceptives

The answer is D. From IM 14.

Ms. Tsvetanova is a 42-year-old female with no significant past medical history presenting to establish care with her primary care physician. On review, she notes a weight gain of 14 kg (30 lbs) over the last three years. She attributes this mostly to her sedentary lifestyle, snacking, and difficulty with portion control. She works as a receptionist for a local physician's office and spends most of her day sitting. She denies constipation, low energy, cold intolerance, muscle weakness, depressed mood, easy bruisability, or other skin changes. On physical exam, vital signs reveal temperature is 36.8 C (98.2 F), pulse is 82 beats/minute, respiratory rate is 12 breaths/minute, blood pressure is 130/82 mmHg, weight is 81.6 kg (180 lbs), and height is 163 cm (64 in). The remainder of her physical exam is normal. Which of the following laboratory tests is most appropriate for the evaluation of this patient? A. 24-hour urine catecholamine levels B. 24-hour urine cortisol level C. Basic metabolic profile D. Lipid profile E. Thyroid stimulating hormone (TSH)

The answer is D. From IM 16.

A 25-year-old fair-skinned female presents with a skin lesion she is concerned about. It is located on her upper-right back. She is a runner and runs outside often. She wears sunscreen in the summer with SPF 30. The lesion has been present for years; however, over the last few months she has noticed a change in size and color. The color was initially solid brown, however, it is now brown with a mixture of darker brown and black. It is not painful and does not itch. She describes no easy bleeding in the area. On physical exam, the lesion is 7 mm, irregular and asymmetric in appearance, and has a mixture of brown and black shades. The lesion is a flat patch without crusting or ulceration. Based on this patient's history and examination, which finding has the highest specificity for melanoma? A. Asymmetric appearance B. Color variation C. Diameter D. Evolving character E. History of frequent sun exposure

The answer is D. From IM 17.

An 84-year-old female comes to the clinic with concern for fatigue for the past six months. Otherwise, a review of systems is negative. On physical examination, her vital signs show pulse is 99 beats/minute, blood pressure is 122/76 mmHg, oxygen saturation is 98% on room air. She appears slightly pale, but is otherwise comfortable. Cardiovascular and pulmonary examination is otherwise normal. Her complete blood count (CBC) is as follows: White blood cell (WBC): 3.2 cells x 103/μL Hemoglobin: 7.5 g/dL Hematocrit (Hct): 22% Mean corpuscular volume (MCV): 84 μm3 Red blood cell distribution width (RDW): 20 fLPlatelet (Plt): 128,000 mm3 Given your concern for a chronic normocytic anemia, you obtain a reticulocyte count, which is 0.05%. What is the next-best diagnostic test at this time? A. Esophagogastroduodenoscopy (EGD) B. Iron studies C. Lactate dehydrogenase (LDH) (serum) D. Peripheral smear E. Thyroid stimulating hormone (TSH)

The answer is D. From IM 19.

A 45-year-old human immunodeficiency virus (HIV)-infected male presents to the emergency department with nonproductive cough, fever, and progressive exertional dyspnea for one week. His CD4 count was 150/microliter one month ago. He has not been adherent to his antiretroviral regimen. On arrival, his temperature is 38.3 C (100.9 F), heart rate is 98 beats/minute, blood pressure is 110/70 mmHg, and oxygen saturation is 89% on room air. Physical exam is unremarkable except for oral thrush. Chest x-ray shows diffuse bilateral infiltrates with early cystic changes. Which organism is most likely causing this patient's acute symptoms? A. Aspergillus fumigatus B. Candida albicans C. Mycobacterium tuberculosis D. Pneumocystis jirovecii E. Streptococcus pneumoniae

The answer is D. From IM 22.

A 56-year-old male presents to the emergency department with shortness of breath with minimal activity for two days. He endorses orthopnea and paroxysmal nocturnal dyspnea. Past medical history is significant for hypertension, hyperlipidemia, and peripheral vascular disease for which he takes aspirin, amlodipine, and atorvastatin. He has a 20-pack-per-year history of smoking. Vital signs are as follows: temperature is 37.1 C (98.7 F), pulse is 102 beats/minute, blood pressure is 180/100 mmHg, body mass index (BMI) is 36 kg/m2 and oxygen saturation is 94% on two liters of air via nasal cannula. Physical exam is significant for crackles at both lung bases, jugular venous pressure of 9 cm of water, S3 gallop at cardiac apex, and bilateral pitting edema to mid-calves. A complete blood count, basic metabolic panel, troponin T assay, and b-type natriuretic peptide level return within normal limits. An electrocardiogram (ECG) shows sinus tachycardia. His chest radiograph is shown. Which of the following is the most likely cause of this patient's symptoms? A. Acute coronary syndrome B. Acute exacerbation of asthma C. Acute exacerbation of chronic obstructive pulmonary disease D. Acute exacerbation of congestive heart failure E. Pulmonary embolism

The answer is D. From IM 22.

A 78-year-old male has been admitted to the hospital after a severe stroke. Past medical history is significant for congestive heart failure, hypertension, and chronic obstructive pulmonary disease (COPD). On day five of his hospitalization, he develops a fever, productive cough, pleuritic chest pain, and shortness of breath. Pulmonary exam reveals bronchial breath sounds, dullness to percussion, increased tactile vocal fremitus, and whispered pectoriloquy in the right-lower lung fields. There are no adventitious sounds. What is the most likely diagnosis? A. Chronic obstructive pulmonary disease exacerbation B. Congestive heart failure exacerbation C. Pleural effusion D. Pneumonia E. Pulmonary embolism

The answer is D. From IM 22.

A 52-year-old female with stage 4 chronic kidney disease, secondary to hypertensive nephropathy, presents to establish care. Her blood pressure is 154/86 mmHg on 12.5mg of hydrochlorothiazide. Her urine protein/creatinine ratio is 1. What is the best treatment for her hypertension? A. Increase the dose of the hydrochlorothiazide and add lisinopril B. Increase the dose of the hydrochlorothiazide and add losartan C. Stop the hydrochlorothiazide and start chlorthalidone D. Stop the hydrochlorothiazide and start lisinopril E. Stop the hydrochlorothiazide and start losartan

The answer is D. From IM 23.

A 59-year-old male with a history of end-stage kidney disease, for which he is on hemodialysis, presents to the emergency department with 12 hours of fever and severe headache. During a hospitalization 10 days ago, he was diagnosed with Staph aureus bacteremia. Since he was discharged five days ago, he has been receiving vancomycin at his dialysis sessions. His exam is notable for a temperature of 39.1C (102.4F), a stiff neck with flexion, and blurred edges of the optic disc on retinal exam. He is fatigued, but his neurological exam is otherwise normal. He was given intravenous (IV) ceftriaxone by the emergency medicine team. Which of the following is a contraindication for performing a lumbar puncture? A. Aspirin, 81 mg daily B. Bacteremia C. Fever D. Papilledema E. Previous administration of antibiotics

The answer is D. From IM 24.

A 75-year-old female is brought to the emergency department with a one-day history of somnolence and confusion. She resides in a skilled nursing facility. Medical history is significant for mild dementia. Two weeks ago she received ciprofloxacin for white blood cells (WBCs) on urinalysis. She is normally alert and able to converse, but for the past day she has been sleeping most of the time with only brief intervals of alertness. She has urge incontinence at baseline, but has previously denied any dysuria, increased frequency, or hematuria. Nursing records show six liquid stools over the past two days, but no fever or chills, and no abdominal or flank pain. On exam, her vital signs are normal. She states the days of the week forward correctly, but cannot recite the days backwards. She falls asleep during your interview. The remainder of her exam, including her abdominal and neurologic exam, are unremarkable. Basic metabolic profile shows a mildly elevated blood urea nitrogen (BUN). A complete blood count (CBC) reveals a mild chronic anemia and leukocytosis. Which of the following would be the most appropriate diagnostic study to determine the cause of her delirium? A. Computed tomography (CT) of her head B. Electroencephalogram (EEG) C. Ferritin D. Stool C. difficile toxin test E. Urine culture

The answer is D. From IM 25.

A 78-year-old male who recently entered an assisted living facility is brought to the office because of increasing confusion and somnolence. He has a history of mild dementia, hypertension, hypercholesterolemia, benign prostatic hypertrophy, and insomnia. He has had no fevers, pain, vomiting, diarrhea, constipation, or dysuria. He takes amlodipine, atorvastatin, donepezil, lorazepam, and tamsulosin. His vital signs are normal, and aside from short-term memory loss, his physical exam is unremarkable for abnormal findings. Which of the following medications should be discontinued? A. Amlodipine B. Atorvastatin C. Donepezil D. Lorazepam E. Tamsulosin

The answer is D. From IM 25.

A 25-year-old female is brought to the emergency department for a one-hour history of decreased mental status. She was found by her roommate barely arousable on the couch with an empty unlabeled pill bottle on the floor next to her. The woman mumbles inaudible responses to questions. Physical examination reveals pinpoint pupils; decreased muscle tone; and slow, shallow respirations. Which of the following medication classes is the most likely cause of this patient's findings? A. Antihistamines B. Barbiturates C. Benzodiazepines D. Opioids E. Tricyclic antidepressants

The answer is D. From IM 26.

A 45-year-old male presents to the emergency department having ingested an unknown liquid found in his garage. He has a severe metabolic acidosis and an osmolar gap of 30. Which of the following findings on urinalysis would be characteristic of ethylene glycol toxicity? A. Calcium carbonate crystals B. Calcium phosphate crystals C. Cystine crystals D. Monohydrate calcium oxalate crystals E. Uric acid crystals

The answer is D. From IM 26.

A 35-year-old female comes to the clinic because of a three-month history of abdominal distension. She has concerns about diffuse abdominal pain, polyuria, and polydipsia. She has lost 15 pounds over the past six months despite not changing her diet. She was previously healthy. Physical exam is significant for a distended abdomen with shifting dullness and a fluid wave. Laboratory studies are shown: Serum calcium: 12.5 mg/dL Parathyroid hormone: 8 pg/mL 1,25(OH)2 Vitamin D: 40 pg/mL 5x3 cm mass in the left adnexal area. Which of the following is the most-likely cause of this patient's findings? A. Direct production of parathyroid hormone (PTH) by the malignant cells B. Direct synthesis of 1,25(OH)2 Vitamin D by the tumor C. Osteoclast activation and release of calcium from an osteolytic tumor D. Parathyroid hormone-related protein activation of osteoclasts E. Parathyroid hormone-related protein increasing renal calcium elimination

The answer is D. From IM 27.

You are evaluating a 53-year-old male in the emergency department who is presenting with a chief concern of shortness of breath. He reports a one-day history of progressively worsening dyspnea on exertion, productive cough, and subjective fever. His past medical history includes chronic obstructive pulmonary disease (COPD) and ischemic cardiomyopathy with a stable ejection fraction (EF) of 40%. Which of the following physical exam findings is most supportive of an infectious etiology of his shortness of breath? A. Absent breath sounds on one side B. Bibasilar crackles and mild wheezes C. Diminished inspiratory effort D. Increased fremitus and egophony over part of the lung E. Tachycardia with normal breath sounds on exam

The answer is D. From IM 28.

xYou are seeing a frail 85-year-old female who was admitted to your service three months ago with a chronic obstructive pulmonary disease (COPD) exacerbation. Despite your tobacco cessation counseling, she continued to smoke after discharge. She returns with shortness of breath. Six days ago, she had sudden worsening of her shortness of breath, and has since felt progressively more breathless, and is now unable to walk more than 30 feet. She can also no longer climb the stairs to her bedroom, so she is sleeping on the living room sofa. She has no cough, fever, or new orthopnea. In addition to a COPD exacerbation, what other illness most likely explains her acute presentation? A. Acute myocardial infarction B. Idiopathic pulmonary fibrosis C. Lung cancer D. Pulmonary embolism E. Tension pneumothorax

The answer is D. From IM 28.

A 24-year-old female is evaluated in the hospital for 8/10 abdominal pain. She was admitted one day ago for cholecystitis, and an open cholecystectomy was performed today. She was previously healthy. Medications are ceftriaxone and acetaminophen. Temperature is 37.5 C (99.5 F), pulse is 100 beats/minute, respiratory rate is 28 breaths/minute, blood pressure is 120/80 mmHg, and oxygen saturation is 95% on room air. On physical exam, she appears uncomfortable and anxious. Her surgical incision from this morning is intact without erythema. She is tender near the surgical incision. Lungs are clear to auscultation bilaterally. Cardiovascular exam reveals no murmur and normal S1 and S2. What is the most appropriate step in management of this patient? A. Arterial blood gas B. Chest radiograph C. Computed tomography pulmonary angiogram (CTPA) D. Intravenous (IV) morphine E. Nebulized albuterol

The answer is D. From IM 29.

You are interviewing a 57-year-old female in the emergency department with unilateral leg swelling. Which of the following are additional risk factors for deep-vein thrombosis (DVT) that you should inquire about in the history? A. Alcohol use of three glasses of wine per day B. Allergy to aspirin C. History of Helicobacter pylori (H. pylori) infection D. History of lupus E. Vegetarian diet

The answer is D. From IM 30.

Your new admission is a 60-year-old male with a massive retroperitoneal hemorrhage which required embolization in interventional radiology. His only past medical history is atrial fibrillation, for which he takes rivaroxaban. His blood pressure was 70/30 mmHg upon arrival in the emergency department (ED) and has since stabilized. His admission labs show his blood urea nitrogen (BUN) is 45 mg/dL and creatinine (Cr) is 3.2 mg/dL. His urine microscopy shows muddy brown casts. What is the most-likely diagnosis? A. Acute glomerulonephritis (Acute GN) B. Acute interstitial nephritis (AIN) C. Acute pyelonephritis D. Acute tubular necrosis (ATN) E. Chronic kidney disease (CKD)

The answer is D. From IM 33.

A 45-year-old male comes to clinic because of a four-day history of increasing back pain and one week of fever, chills, fatigue, and generalized malaise. Medical history is significant for a T10 spinal cord injury causing paraplegia following a motor vehicle accident 10 years ago. He has required self-catheterization for bladder decompression since the accident. Temperature is 38.7 C (101.7 F), pulse is 102 beats/minute, respiratory rate is 20 breaths/minute, and blood pressure is 98/55 mmHg. Oxygen saturation is 99% on room air. He appears tired and slightly pale. He has mild tenderness to palpation over the thoracic spine between his shoulder blades. His bladder is palpable in his lower abdomen. He has loss of sensation at the level of his umbilicus and paralysis of his lower extremities with increased tone. The remainder of his physical examination is normal. Which of the following is the most likely cause of this patient's back pain? A. Abdominal aortic aneurysm B. Cauda equina syndrome C. Infective endocarditis D. Spinal abscess E. Urinary tract infection

The answer is D. From IM 34.

A 37-year-old female comes to the clinic with a two-week history of intermittent fevers to 39.4 C (103 F). She also complains of chills, myalgias, and headaches associated with the fevers which occur about twice a day. She was previously healthy. She lives in the northeast United States, but traveled to rural Pakistan one month ago to visit family. Laboratory studies are shown as: White blood count (WBC): 8500 /mm3 Hemoglobin: 9.2 g/dl Hematocrit (Hct): 28% Platelets: 75000 /mm3. Which diagnostic study will most-likely confirm the etiology of her symptoms? A. Chest x-ray B. Computed tomography (CT) of the head C. Lyme enzyme immunoassay (EIA) D. Thick and thin blood smears E. Tuberculosis skin testing

The answer is D. From IM 35.

A 59-year-old female presents to the clinic with a four-week history of intermittent fever, malaise, and decreased appetite. One week ago, she went to the emergency department (ED) for evaluation of a new left-sided throbbing frontal headache and pain with chewing. She brings in a report of labs from the ED visit, showing a white blood cell (WBC) count of 8400 cells/mcL, hemoglobin of 11 gm/dL, platelets of 472,000 plt/mcL, a normal urinalysis, normal complete metabolic panel, and non-contrast head computed tomography (CT) report that shows no significant findings. She reports she was sent home with a prescription for sumatriptan to treat her headache and instructions to follow up with her primary care provider. Which of the following additional history questions would be most relevant in helping you differentiate among the etiologies for fever in this patient? A. Asian ancestry B. Influenza vaccination status C. Migraines as a teenager D. Vision loss

The answer is D. From IM 35.

A 49-year-old male comes into the clinic with new abdominal swelling. He has no past medical history and had otherwise been well prior to presentation. On exam, he is well-appearing and in no apparent distress. His vital signs, cardiac, and pulmonary exams are normal. His abdomen is distended and tense with a positive fluid wave. He is not tender on exam. The clinic is equipped with an ultrasound, so you prepare to do a diagnostic paracentesis. Before you start, you obtain the patient's consent. Which of the following is NOT a component of informed consent? A. Ability to find the cause of the ascites B. Bleeding risks of a paracentesis C. Infection risks of a paracentesis D. Size of the needle used in the procedure E. Watchful waiting as an alternative to the procedure

The answer is D. From IM 36.

A 55-year-old male presents with acute, severe, substernal chest pressure. He has a history of peptic ulcer disease. He drinks two beers daily and smokes a half-a-pack of cigarettes daily. He has multiple family members with inflammatory bowel disease. He works long hours outdoors in construction. Electrocardiogram (ECG) confirms an ST-elevation myocardial infarction (STEMI). Which of the following is a cardiac risk factor in this patient? A. Construction work B. Drinking two beers a day C. Having a family history of inflammatory bowel disease D. Having peptic ulcer disease E. Smoking half-a-pack of cigarettes a day

The answer is E. From IM 01.

A 52-year-old female presents with one month of progressive dyspnea on exertion, a productive cough of clear sputum, bilateral leg swelling, and orthopnea. She has had no chest pain or fevers. She has a history of poorly controlled hypertension and 30-pack-years of smoking. Before beginning her physical exam, you determine that her pre-exam probability of heart failure is moderate. On exam, you find a jugular venous pulse and pressure (JVP) of 12 cm with positive hepatojugular reflux, regular heart rate, normal S1 and S2 with S3 gallop, 3/6 systolic crescendo-decrescendo murmur at the right upper sternal border, crackles at both lung bases. Which of the following physical examination findings most increases your post-exam probability of congestive heart failure? A. Bibasilar crackles B. Hepatojugular reflux C. Increased jugular venous pressure D. Systolic murmur in the aortic area E. Third heart sound

The answer is E. From IM 04.

A 56-year-old female with non-ischemic cardiomyopathy and hypertension presents to the office for a routine followup. Her last hospitalization for a congestive heart failure (CHF) exacerbation was two years ago. Currently, she has no shortness of breath, orthopnea, leg edema, or chest pain. She has been following a low-salt diet and does not drink alcohol. Her medications are carvedilol and a baby aspirin. Her home blood pressure measurements have ranged from 140-150/80-90 mmHg. Her exam is notable for a blood pressure of 150/90 mmHg. Her pulse is 60 beats/minute with normal S1 and S2 with no murmurs or gallops, and she has a normal respiratory rate. Her jugular venous pulse and pressure (JVP) is normal and her lungs are clear. Her point of maximal impulse (PMI) is laterally displaced. What medication should you add that can also improve her mortality from heart failure? A. Amlodipine B. Digoxin C. Furosemide D. Hydrochlorothiazide E. Ramipril

The answer is E. From IM 04.

A 63-year-old male comes to you for a new-patient visit. He has a past medical history of a myocardial infarction (MI) and hyperlipidemia one year ago. He ran out of his previous medications several weeks ago and cannot remember what they were. He is asymptomatic and has a normal physical exam except for a blood pressure of 165/92 mmHg. His labs come back with a hemoglobin A1c of 5.4%, an elevated LDL, and normal electrolytes and renal function. In addition to a statin and daily aspirin, what other medication would be the best to start him on? A. Amlodipine B. Diltiazem C. Hydrochlorothiazide D. Lisinopril E. Metoprolol

The answer is E. From IM 06.

Mr. Sands is a 63-year-old male with a 25-year history of hypertension and type 2 diabetes mellitus who presents to the office for a follow-up of his blood pressure. Given Mr. Sands' long-standing history of poor blood pressure control, you evaluate him for end-organ effects of long-standing hypertension. Which of the following findings would be most specific for possible end-organ damage from hypertension? A. Decreased monofilament sensation on bilateral feet B. Left basilar crackles on lung exam C. Left leg edema D. Paralysis of upper and lower face on right side E. Point of maximum impulse (PMI) of the heart palpated 6 cm lateral to the mid-clavicular line

The answer is E. From IM 06.

A 34-year-old female presents to the hospital for one day of periumbilical abdominal pain that is 8/10 in severity. It radiates to her back and is associated with severe nausea and vomiting. She does not drink alcohol. She has had three Caesarean sections, gastric bypass surgery for weight loss, and a tubal ligation. On exam, her vitals show temperature is 37 C (98.6 F), pulse is 100 beats/minute, respiratory rate is 18 breaths/minute, and blood pressure is 130/70 mmHg. She has decreased bowel sounds on auscultation. On palpation, she has periumbilical tenderness to palpation and voluntary guarding. Her complete blood count (CBC) and comprehensive metabolic panel are normal. Her lipase is 520 U/L, and her urine pregnancy test is negative. She is given a liter of intravenous (IV) normal saline and has normal saline running at 250 cc/hour. Her pain is controlled with morphine. What is the next step in her evaluation? A. Computed tomography (CT) of the abdomen and pelvis B. Endoscopic retrograde cholangiopancreatography (ERCP) C. Hepatobiliary (HIDA) scan D. Magnetic resonance cholangiopancreatography (MRCP) E. Right-upper-quadrant ultrasound (RUQ US)

The answer is E. From IM 09.

Ms. Azarola is a 46-year-old female with a past surgical history of an exploratory laparotomy for endometriosis, two Caesarean sections, and a tubal ligation. She presents to the emergency department with eight hours of intermittent, diffuse, severe abdominal pain with nausea and vomiting. On physical exam, her vital signs show temperature is 37.2 C (99 F), pulse is 110 beats/minute, respiratory rate is 18 breaths/minute, and blood pressure is 148/90 mmHg. Her abdomen reveals high-pitched bowel sounds and moderate distension. There is a hollow sound when you tap on her abdomen. On palpation, she has guarding, but no rigidity. An abdominal x-ray reveals air fluid levels and distension in the small bowel with minimal gas in the large intestine. You place orders for routine labs, non per os (NPO), intravenous (IV) fluids, and insert a nasogastric tube, placing it to suction. What should your next action be? A. Barium enema B. Follow-up abdominal plain film C. Nutrition consultation D. Right-upper-quadrant ultrasound E. Surgical consultation

The answer is E. From IM 09.

A 43-year-old healthy female comes to the clinic to discuss a cancer screening. Her family history is significant for a first cousin with colon cancer diagnosed at 48 years of age. There is no history of cancer in her immediate family. She does not smoke or consume alcohol, and she exercises three times a week. Vital signs show her pulse is 68 beats/minute, blood pressure is 132/72 mmHg, and body mass index (BMI) is 23.8 kg/m2. Physical examination is normal. With regards to colon cancer screening, what would you recommend for this patient? A. Computed tomography (CT) colonography now and then every 5 years B. Fecal immunochemical testing now and then every year C. Fecal occult blood testing at age 50 and then every 3 years D. Flexible sigmoidoscopy now and then every five years E. Routine colonoscopy at age 50 and then every 10 years

The answer is E. From IM 13.

A 45-year-old female presents with urgency, frequency, and pain when she urinates. She also has left-lower back pain, fever, chills, nausea, and vomiting. She has a history of type 2 diabetes mellitus and takes metformin. On physical examination, her vital signs show her temperature 38.9 C (102 F), blood pressure is 90/55 mmHg, pulse is 120 beats/minute, respiratory rate is 18 breaths/minute, and oxygen saturation is 98% on ambient air. She has left flank tenderness and lower abdominal tenderness. Laboratory studies showed an elevated white blood cell count. Urinalysis showed pyuria with white blood cell cast. Blood and urine cultures were sent. She is admitted to the hospital and started on intravenous fluids and ceftriaxone. Urine culture shows pan-sensitive E. coli. Despite three days of antibiotics, she remains febrile with a peak temperature of 38.9 C (102 F). Her abdominal pain persists, albeit is slightly better, and she continues to have nausea and vomiting. Which of the following is the most appropriate next step? A. Add vancomycin B. Change ceftriaxone to another class of antibiotic C. Consult urology D. Obtain a chest radiograph E. Obtain a renal ultrasound

The answer is E. From IM 14.

A 45-year-old female presents with a 12-day history of fever to 39.2 C (102.6 F), unilateral face and tooth pain, a frontal headache, and purulent nasal discharge. She has no chronic medical conditions. On examination, temperature is 38.4 C (101.1 F), pulse is 96 beats/minute, respiratory rate is 16 breaths/minute, and blood pressure is 116/74 mmHg. There is tenderness to palpation over the left maxillary sinus. Nasal examination shows inflamed turbinates bilaterally with purulent discharge on the left. The remainder of the exam is normal. What is the most-likely pathogen causing her symptoms? A. Adenovirus B. Moraxella catarrhalis C. Parainfluenza virus D. Staphylococcus aureus E. Streptococcus pneumoniae

The answer is E. From IM 15.

In mid-September, a 52-year-old male presents with a three-day history of sore throat, nasal congestion, rhinorrhea, dry cough, and fatigue. He has no chronic medical conditions and does not smoke. On physical examination, vital signs are normal. There is mild, clear nasal discharge with no sinus tenderness. The oropharynx is without injection or exudate. There is no lymphadenopathy. Tympanic membranes are dull bilaterally and a small effusion is noted on the left. Lungs are clear to auscultation. What is the most likely pathogen causing his symptoms? A. Adenovirus B. Coronavirus C. Parainfluenza virus D. Respiratory syncytial virus E. Rhinovirus

The answer is E. From IM 15.

A 75-year-old male presents to the clinic for the follow-up of a skin biopsy of a nevus. The pathology report says that histology is consistent with a junctional nevus. Which of the following would be the expected histologic findings in this patient? A. Atypical melanocytes B. Hemosiderin deposition C. Melanocytes primarily within the dermis D. Melanocytes located in both the dermis and dermal-epidermal junction E. Nests of melanocytes at the dermal-epidermal junction

The answer is E. From IM 17.

A 45-year-old female comes to your clinic with concerns of fatigue and shortness of breath on exertion for the past three weeks. She has a history of hypertension and obesity, and underwent gastric bypass surgery five years ago. She stopped taking her medications three years ago due to financial reasons. On physical examination, her pulse is 89 beats/minute and her blood pressure is 100/50 mmHg. She has conjunctival pallor, cheilosis, and glossitis. Cardiac examination reveals a systolic murmur over the mitral area. Neurological examination reveals loss of proprioception. What is the most likely cause of this patient's symptoms? A. Folic acid deficiency B. Hemolytic anemia C. Hypothyroidism D. Iron deficiency anemia E. Vitamin B12 deficiency

The answer is E. From IM 19.

A 33-year-old male with a history of human immunodeficiency virus (HIV) from sexual contact presents with one day of fever, cough productive of green sputum, and a left-lower-lobe infiltrate on chest radiograph. His CD4 count is 440 cells/mm3. What is the most-likely cause of his pneumonia? A. Adenovirus B. Methicillin-resistant Staphylococcus aureus C. Mycobacterium avium complex D. Pneumocystis jiroveci E. Streptococcus pneumoniae

The answer is E. From IM 20.

A 38-year-old female with human immunodeficiency virus (HIV) infection presents to her primary care doctor with concern for daily intermittent fevers to 38.6 C (101.5 F) for four weeks. She also has night sweats and some dyspnea, especially when hiking near her home in Louisville, Kentucky. She doesn't have any additional symptoms. She reports no recent dental work or travel. She has no pets. Her physical exam is unremarkable. Laboratory testing reveals a CD4 count of 220 cells/mcl, normal basic metabolic panel, and a complete blood count (CBC) remarkable for stable mild thrombocytopenia (125,000 mm3). Bacterial blood cultures demonstrate no growth after five days. Chest x-ray shows diffuse pulmonary infiltrates. Which of the following would be the most appropriate test to order for this patient? A. Echocardiogram B. Funduscopic examination C. Lymph node biopsy D. Serum cryptococcal antigen E. Serum histoplasma antigen

The answer is E. From IM 20.

A 33-year-old previously healthy female is admitted to the hospital with pneumonia. Two weeks prior to admission, she developed fever and myalgias. She was diagnosed with influenza at an urgent care clinic. Five days ago, she developed a productive cough, fever, and chills. Today, her temperature is 38.9 C (102 F), pulse is 110 beats/minute, respiration rate is 22 breaths/minute, blood pressure is 90/68 mmHg, and pulse oximetry is 89% on room air. Respiratory exam reveals crackles at the lung bases bilaterally. White blood cell count is 3,400/mm3. The chest x-ray shows multilobular cavitating alveolar infiltrates. Which of the following organisms is the most likely cause of pneumonia in this patient? A. Adenovirus B. Legionella pneumophila C. Mycoplasma pneumoniae D. Pseudomonas aeruginosa E. Staphylococcus aureus

The answer is E. From IM 22.

A 66-year-old female is evaluated in the emergency department with a two-day history of right-sided pleuritic chest pain and intermittent shortness of breath. Past medical history is significant for hypertension, chronic kidney disease, and depression. Medications include lisinopril, baby aspirin, and oral hormone replacement therapy. Physical examination shows her temperature is 37.8 C (100.1 F), pulse is 112 beats/minute, respiratory rate is 22 breaths/minute, blood pressure is 158/76 mmHg, body mass index (BMI) is 32 kg/m2 and oxygen saturation is 89% on room air. Her lungs are clear and heart sounds are normal. Her left leg is edematous and tender to palpation. An electrocardiogram (ECG) shows no ST or T wave changes. Chest x-ray is normal. Laboratory studies are unremarkable except for a creatinine of 2.1 mg/dL. Which of the following is the most appropriate diagnostic test at this time? A. Computed tomography (CT) of the chest B. D-dimer test C. Troponin T test D. Venous ultrasonography of left leg E. Ventilation perfusion (V/Q) scan

The answer is E. From IM 22.

A 63-year-old male with stage 3 chronic kidney disease (CKD), secondary to hypertension, presents for a routine follow-up. His blood pressure is 134/72 mmHg. His physical exam is within normal limits. His labs are significant for a potassium of 5.0 mEq/L and phosphorus of 5 mg/dL. What is the explanation for his hyperkalemia and hyperphosphatemia? A. Excess dietary potassium and phosphorus B. Excess intestinal absorption of potassium and phosphorus C. Impaired intestinal absorption of potassium and phosphorus D. Impaired renal absorption of potassium and phosphorus E. Impaired renal excretion of potassium and phosphorus

The answer is E. From IM 23.

A 21-year-old college student presents to the hospital with one day of fever, headache, and neck stiffness. His retinal and neurological examinations are normal. Ceftriaxone is given, and a lumbar puncture is performed which shows the following: Cloudy fluid White blood cell (WBC): 2,400; 85% polymorphonuclear cells (PMNs) Red blood cell (RBC): 2 mg/dL Protein: 250 mg/dL Glucose: 32 mg/dL. Gram stain: Gram-negative diplococci Which finding in the man's cerebrospinal fluid (CSF) is most specific for bacterial infection? A. Cloudy fluid B. High CSF protein C. Low CSF glucose (hypoglycorrhachia) D. Pleocytosis with neutrophil predominance E. Positive Gram stain

The answer is E. From IM 24.

Which of the following measures is most important when entering the hospital room of a patient who is on droplet precautions? A. Gloves B. N95 respirator mask C. Paper or plastic gown covering the entire body D. Protective eyewear E. Simple mask covering the nose and mouth

The answer is E. From IM 24.

A 45-year-old male with a history of alcohol use disorder is admitted to the hospital with delirium. His brother, with whom he lives, states that the patient normally drinks a 12-pack of beer per day, but for the past two weeks has been trying to quit by drinking a large glass of water every time he craved a beer. Vital signs are normal. On exam, his mucous membranes are moist, lungs are clear, and jugular venous pulse and pressure (JVP) is normal. He is alert and oriented to self and place, but not to time. He can follow one-step commands but is unable to name the days of the week forward or backward. His serum sodium is 120 mEq/L. His urine sodium is 5 mEq/L, and urine osmolarity is low. Which of the following is the best approach to treat his hyponatremia? A. Dextrose in water (D5W) at 100 cc/hour B. Hypertonic (3%) saline intravenous (IV) at 100 cc/hour C. Hypotonic (0.45%) saline IV at 100 cc/hour D. Normal (0.9%) saline IV at 100 cc/hour E. No IV fluids

The answer is E. From IM 25.

You admit a 65-year-old male with a severe chronic obstructive pulmonary disease (COPD) exacerbation. He has a 40-pack-per-year smoking history, and quit five years ago. He has not followed up with primary care appointments in the past, so you take the opportunity to ensure he is up-to-date with preventive measures. Which of the following preventive measures should be addressed at this time? A. Dual energy x-ray absorptiometry (DEXA) scan B. Haemophilus influenzae type b (Hib) vaccination C. Hepatitis A vaccination D. Sputum cytology E. Streptococcus pneumoniae vaccination

The answer is E. From IM 28.

A 73-year-old male with a history of a recent diverticular bleed causing hemodynamic instability presents with right knee pain. He states that he was diagnosed with osteoarthritis of the knee in the past. He has tried acetaminophen, without improvement. He asks what other options are available to help control his pain and improve his mobility. What is the best initial option to treat this patient? A. Glucosamine and chondroitin B. Ibuprofen C. Naproxen D. Opioids E. Physical therapy

The answer is E. From IM 31.

You are in the rheumatology clinic seeing a 32-year-old female with a recent diagnosis of systemic lupus erythematosus (SLE). She informs you that she is G3P0A3, having had three early (<10 weeks gestation) spontaneous abortions. Which of the following antibody tests is most strongly associated with an increased incidence of spontaneous abortions? A. Anti-histidyl-tRNA synthetase (anti-Jo-1) B. Anti-Ro/anti-SSA C. Antineutrophil cytoplasmic (ANCA) D. Antinuclear (ANA) E. Antiphospholipid (APLA)

The answer is E. From IM 32.

It is the Monday after a major national holiday, and you are seeing a 58-year-old male in the emergency department with progressive shortness of breath and generalized weakness over the previous two days. He has a history of end-stage kidney disease (ESKD) due to diabetes and hypertension. He is chronically anuric and undergoes hemodialysis (HD) every Tuesday, Thursday, and Saturday. He says, "You know, Doc, I definitely indulged a bit too much at the holiday dinner." On exam, he appears short of breath, sitting upright on the gurney and speaking in 2- to 3-word sentences. His temperature is 36.1 C (97 F), pulse is 75 beats/minute, respiratory rate is 20 breaths/minute, blood pressure is 180/60 mmHg, and oxygen saturation is 94% on 10 liters/minute face mask. His exam is notable for diffuse crackles bilaterally, jugular venous pulse and pressure (JVP) of 12 cm while upright, lower extremity pitting edema to above his knees bilaterally, and a left arteriovenous (AV) fistula with a strong thrill. His labs are notable for the following: Sodium (Na): 132 mEq/L Potassium (K): 5.5 mEq/L Chloride (Cl): 111 mEq/L Bicarbonate (HCO3): 19 mEq/L Blood urea nitrogen (BUN): 48 mg/dL Creatinine (Cr): 5.9 mg/dL Phosphorus: 5.1 mg/dL Albumin: 3.4 g/dL Troponin: 0.04 ng/mL pH: 7.35 Chest x-ray shows pulmonary vascular congestion and pulmonary edema. Electrocardiogram (ECG) is normal. What is the next-best step in evaluation and/or management of this patient? A. Blood cultures and intravenous (IV) antibiotics B. Calcium gluconate IV C. Computed tomography angiography (CTA) of the chest D. Furosemide 40 mg IV push and reevaluation in four hours E. Nephrology consult for urgent hemodialysis

The answer is E. From IM 33.

A 50-year-old male presents to the office with concerns of low back pain. He has a five-year history of hypertension which is well controlled on hydrochlorothiazide. A physical exam reveals an uncomfortable obese male whose pulse is 90 beats/minute, respiratory rate is 22 breaths/minute, and blood pressure is 140/85 mmHg. The back pain is reproduced by lifting the right leg 45 degrees from the horizontal table while he is reclined in the supine position. Right-posterior calf pain which radiates down to the foot also occurs with this maneuver. Which additional physical exam finding would help confirm the diagnosis? A. Seated bilateral-leg raise reproducing the pain at 25-degrees extension B. Seated left-leg raise reproducing the pain at 25-degrees extension C. Seated left-leg raise reproducing the pain at 35-degrees extension D. Seated right-leg raise reproducing the pain at 25-degrees extension E. Seated right-leg raise reproducing the pain at 35-degrees extension

The answer is E. From IM 34.

A 60-year-old male presents with two months of progressive, focal pain over his mid-back. He has no neurologic symptoms. He also notes night sweats, unintended weight loss of 10 pounds, and general fatigue. His exam is notable for focal tenderness to palpation over the T10 vertebra. You order magnetic resonance imaging (MRI), which shows lesions of the T9 and T10 vertebra, concerning for metastases. Which of the following malignancies is most likely to cause focal spine pain in this patient? A. Colon B. Esophagus C. Kidney D. Liver E. Prostate

The answer is E. From IM 34.

A 49-year-old female returns to the emergency department (ED) with a five-week history of fever, night sweats, decreased appetite, weight loss, and dry cough. She has a history of human immunodeficiency virus (HIV) and hyperlipidemia and has been off medications for more than one year since she was released from prison. Two weeks ago, she went to the ED for these symptoms. Labs from the ED visit revealed a white blood cell (WBC) count of 10,600 cells/mm3, a CD4 count of 90 cells/mm3, hemoglobin of 11,100cells/mm3, platelets of 492,000 plt/mm3, a normal urinalysis (UA), normal complete metabolic panel, negative blood cultures, and a chest x-ray with moderate pleural effusions with mediastinal lymphadenopathy. Hospital admission was recommended, but she left against medical advice. Which of the following is the next best step in evaluating this patient? A. Biopsy the mediastinal lymph nodes B. Obtain blood cultures C. Send pleural fluid for analysis and culture D. Send serum for an interferon gamma-release assay (IGRA) E. Send sputum sample for an acid-fast bacillus (AFB) smear

The answer is E. From IM 35.

A 35-year-old female presents with two days of constant, sharp, left-sided chest pain. There was no clear precipitant for her pain, and she does not identify any alleviating or aggravating factors. On physical exam, she appears comfortable and has normal vital signs. Cardiac exam demonstrates regular rate and rhythm, normal S1 and S2, and no murmurs. She has point tenderness over the left-upper-sternal border. Electrocardiogram (ECG) showed normal sinus rhythm with no ST or T wave changes. What is the most likely diagnosis? A. Acute coronary syndrome B. Costochondritis C. Panic disorder D. Pneumothorax E. Pulmonary embolism

There is no best answer.... But I put costochondritis.


Conjuntos de estudio relacionados

Senate, House, Both, or Neither?

View Set

Mastering Math Facts Multiplication Set J (9x7, 7x9, 5x5)

View Set

WGU C955 - Module 2: Fractions, Decimals, & Percentages

View Set

Chapter 38:Vehicle Extrication and Special Rescue

View Set

Pharmacology A ATI Audio Friendly

View Set

Fundamental concepts of chem ch 4

View Set